Разное

Пересечение круги эйлера примеры: Круги эйлера примеры решения. Отношения между понятиями. круги эйлера. Изучение нового материала

Содержание

Круги эйлера примеры решения. Отношения между понятиями. круги эйлера. Изучение нового материала

Если вы думаете, что ничего не знаете о кругах Эйлера, вы ошибаетесь. На самом деле вы наверняка не раз с ними сталкивались, просто не знали, как это называется. Где именно? Схемы в виде кругов Эйлера легли в основу многих популярных интернет-мемов (растиражированных в сети изображений на определенную тему).

Давайте вместе разберемся, что же это за круги, почему они так называются и почему ими так удобно пользоваться для решения многих задач.

Происхождение термина

– это геометрическая схема, которая помогает находить и/или делать более наглядными логические связи между явлениями и понятиями. А также помогает изобразить отношения между каким-либо множеством и его частью.

Пока не очень понятно, верно? Посмотрите на этот рисунок:

На рисунке представлено множество – все возможные игрушки. Некоторые из игрушек являются конструкторами – они выделены в отдельный овал. Это часть большого множества «игрушки» и одновременно отдельное множество (ведь конструктором может быть и «Лего», и примитивные конструкторы из кубиков для малышей). Какая-то часть большого множества «игрушки» может быть заводными игрушками. Они не конструкторы, поэтому мы рисуем для них отдельный овал. Желтый овал «заводной автомобиль» относится одновременно к множеству «игрушки» и является частью меньшего множества «заводная игрушка». Поэтому и изображается внутри обоих овалов сразу.

Ну что, так стало понятнее? Именно поэтому круги Эйлера – это тот метод, который наглядно демонстрирует: лучше один раз увидеть, чем сто раз услышать. Его заслуга в том, что наглядность упрощает рассуждения и помогает быстрее и проще получить ответ.

Автор метода – ученый Леонард Эйлер (1707-1783). Он так и говорил о названных его именем схемах: «круги подходят для того, чтобы облегчить наши размышления». Эйлер считается немецким, швейцарским и даже российским математиком, механиком и физиком. Дело в том, что он много лет проработал в Петербургской академии наук и внес существенный вклад в развитие российской науки.

До него подобным принципом при построении своих умозаключений руководствовался немецкий математик и философ Готфрид Лейбниц.

Метод Эйлера получил заслуженное признание и популярность. И после него немало ученых использовали его в своей работе, а также видоизменяли на свой лад. Например, чешский математик Бернард Больцано использовал тот же метод, но с прямоугольными схемами.

Свою лепту внес также немецкий математике Эрнест Шредер. Но главные заслуги принадлежат англичанину Джону Венну. Он был специалистом в логике и издал книгу «Символическая логика», в которой подробно изложил свой вариант метода (использовал преимущественно изображения пересечений множеств).

Благодаря вкладу Венна метод даже называют диаграммами Венна или еще Эйлера-Венна.

Зачем нужны круги Эйлера?

Круги Эйлера имеют прикладное назначение, то есть с их помощью на практике решаются задачи на объединение или пересечение множеств в математике, логике, менеджменте и не только.

Если говорить о видах кругов Эйлера, то можно разделить их на те, что описывают объединение каких-то понятий (например, соотношение рода и вида) – мы их рассмотрели на примере в начале статьи.

А также на те, что описывают пересечение множеств по какому-то признаку. Таким принципом руководствовался Джон Венн в своих схемах. И именно он лежит в основе многих популярных в интернете мемов. Вот вам один из примеров таких кругов Эйлера:

Забавно, правда? И главное, все сразу становится понятно. Можно потратить много слов, объясняя свою точку зрения, а можно просто нарисовать простую схему, которая сразу расставит все по местам.

Кстати, если вы не можете определиться, какую профессию выбрать, попробуйте нарисовать схему в виде кругов Эйлера. Возможно, чертеж вроде этого поможет вам определиться с выбором:

Те варианты, которые окажутся на пересечении всех трех кругов, и есть профессия, которая не только сможет вас прокормить, но и будет вам нравиться.

Решение задач с помощью кругов Эйлера

Давайте рассмотрим несколько примеров задач, которые можно решить с помощью кругов Эйлера.

Вот на этом сайте – http://logika.vobrazovanie.ru/index.php?link=kr_e.html Елена Сергеевна Саженина предлагает интересные и несложные задачи, для решения которых потребуется метод Эйлера. Используя логику и математику, разберем одну из них.

Задача про любимые мультфильмы

Шестиклассники заполняли анкету с вопросами об их любимых мультфильмах. Оказалось, что большинству из них нравятся «Белоснежка и семь гномов», «Губка Боб Квадратные Штаны» и «Волк и теленок». В классе 38 учеников. «Белоснежка и семь гномов» нравится 21 ученику. Причем трем среди них нравятся еще и «Волк и теленок», шестерым – «Губка Боб Квадратные Штаны», а один ребенок одинаково любит все три мультфильма. У «Волка и теленка» 13 фанатов, пятеро из которых назвали в анкете два мультфильма. Надо определить, скольким же шестиклассникам нравится «Губка Боб Квадратные Штаны».

Решение:

Так как по условиям задачи у нас даны три множества, чертим три круга. А так как по ответам ребят выходит, что множества пересекаются друг с другом, чертеж будет выглядеть так:

Мы помним, что по условиям задачи среди фанатов мультфильма «Волк и теленок» пятеро ребят выбрали два мультфильма сразу:

Выходит, что:

21 – 3 – 6 – 1 = 11 – ребят выбрали только «Белоснежку и семь гномов».

13 – 3 – 1 – 2 = 7 – ребят смотрят только «Волк и теленок».

Осталось только разобраться, сколько шестиклассников двум другим вариантам предпочитает мультфильм «Губка Боб Квадратные Штаны». От всего количества учеников отнимаем всех тех, кто любит два других мультфильма или выбрал несколько вариантов:

38 – (11 + 3 + 1 + 6 + 2 + 7) = 8 – человек смотрят только «Губка Боб Квадратные Штаны».

Теперь смело можем сложить все полученные цифры и выяснить, что:

мультфильм «Губка Боб Квадратные Штаны» выбрали 8 + 2 + 1 + 6 = 17 человек. Это и есть ответ на поставленный в задаче вопрос.

А еще давайте рассмотрим задачу , которая в 2011 году была вынесена на демонстрационный тест ЕГЭ по информатике и ИКТ (источник – http://eileracrugi.narod.ru/index/0-6).

Условия задачи:

В языке запросов поискового сервера для обозначения логической операции «ИЛИ» используется символ «|», а для логической операции «И» – символ «&».

В таблице приведены запросы и количество найденных по ним страниц некоторого сегмента сети интернет.

ЗапросНайдено страниц (в тысячах)
Крейсер | Линкор7000
Крейсер4800
Линкор4500

Какое количество страниц (в тысячах) будет найдено по запросу Крейсер & Линкор ?

Считается, что все вопросы выполняются практически одновременно, так что набор страниц, содержащих все искомые слова, не изменялся за время выполнения запросов.

Решение:

При помощи кругов Эйлера изобразим условия задачи. При этом цифры 1, 2 и 3 используем, чтобы обозначить полученные в итоге области.

Опираясь на условия задачи, составим уравнения:

  1. Крейсер | Линкор: 1 + 2 + 3 = 7000
  2. Крейсер: 1 + 2 = 4800
  3. Линкор: 2 + 3 = 4500

Чтобы найти Крейсер & Линкор (обозначенный на чертеже как область 2), подставим уравнение (2) в уравнение (1) и выясним, что:

4800 + 3 = 7000, откуда получаем 3 = 2200.

Теперь этот результат мы можем подставить в уравнение (3) и выяснить, что:

2 + 2200 = 4500, откуда 2 = 2300.

Ответ: 2300 – количество страниц, найденных по запросу Крейсер & Линкор.

Как видите, круги Эйлера помогают быстро и просто решить даже достаточно сложные или просто запутанные на первый взгляд задачи.

Заключение

Полагаю, нам удалось убедить вас, что круги Эйлера – не просто занимательная и интересная штука, но и весьма полезный метод решения задач. Причем не только абстрактных задач на школьный уроках, но и вполне себе житейских проблем. Выбора будущей профессии, например.

Вам еще наверняка будет любопытно узнать, что в современной массовой культуре круги Эйлера нашли отражение не только в виде мемов, но и в популярных сериалах. Таких, как «Теория большого взрыва» и «4исла».

Используйте это полезный и наглядный метод для решения задач. И обязательно расскажите о нем друзьям и одноклассникам. Для этого под статьей есть специальные кнопки.

сайт, при полном или частичном копировании материала ссылка на первоисточник обязательна.

Каждый предмет или явление обладает некими свойствами (признаками).

Получается, что составить понятие об объекте означает, прежде всего, умение отличить его от других сходных с ним объектов.

Можно сказать, что понятие – это мысленное содержание слова.

Понятие – это форма мысли, отображающая предметы в их наиболее общих и существенных признаках.

Понятие – это форма мысли, а не форма слова, так как слово лишь метка, которой мы помечаем ту или иную мысль.

Слова могут быть различны, но при этом обозначать одно и то же понятие. По-русски – «карандаш», по-английски – «pencil», по-немецки – bleistift. Одна и та же мысль в разных языках имеет разное словесное выражение.

ОТНОШЕНИЯ МЕЖДУ ПОНЯТИЯМИ. КРУГИ ЭЙЛЕРА.

Понятия, имеющие в своих содержаниях общие признаки, называются СРАВНИМЫМИ («адвокат» и «депутат»; «студент» и «спортсмен»).

В противном случае, понятия считаются НЕСРАВНИМЫМИ («крокодил» и «блокнот»; «человек» и «пароход»).

Если кроме общих признаков понятия имеют и общие элементы объёма, то они называются СОВМЕСТИМЫМИ .

Существует шесть видов отношений между сравнимыми понятиями. Отношения между объёмами понятий удобно обозначать с помощью кругов Эйлера (круговые схемы, где каждый круг обозначает объём понятия).

ВИД ОТНОШЕНИЯ МЕЖДУ ПОНЯТИЯМИИЗОБРАЖЕНИЕ С ПОМОЩЬЮ КРУГОВ ЭЙЛЕРА
РАВНОЗНАЧНОСТЬ (ТОЖДЕСТВЕННОСТЬ) Объёмы понятий полностью совпадают. Т.е. это понятия, которые различаются по содержанию, но в них мыслятся одни и те же элементы объёма. 1) А – Аристотель В – основатель логики 2) А – квадрат В – равносторонний прямоугольник
ПОДЧИНЕНИЕ (СУБОРДИНАЦИЯ) Объём одного понятия полностью входит в объём другого, но не исчерпывает его. 1) А – человек В – студент 2) А – животное В – слон
ПЕРЕСЕЧЕНИЕ (ПЕРЕКРЕЩИВАНИЕ) Объёмы двух понятий частично совпадают. То есть понятия содержат общие элементы, но и включают элементы, принадлежащие только одному из них. 1) А – юрист В – депутат 2) А – студент В – спортсмен
СОПОДЧИНЕНИЕ (КООРДИНАЦИЯ) Понятия, не имеющие общих элементов, полностью входят в объём третьего, более широкого понятия. 1) А – животное В – кот; С – собака; D – мышь 2) А – драгоценный металл В – золото; С – серебро; D – платина
ПРОТИВОПОЛОЖНОСТЬ (КОНТРАРНОСТЬ) Понятия А и В не просто включены в объём третьего понятия, а как бы находятся на его противоположных полюсах. То есть, понятие А имеет в своём содержании такой признак, которых в понятии В заменён на противополжный. 1) А – белый кот; В – рыжий кот (коты бывают и чёрными и серыми) 2) А – горячий чай; холодный чай (чай может быть и тёплым) Т.е. понятия А и В не исчерпывают всего объёма понятия, в которое они входят.
ПРОТИВОРЕЧИЕ (КОНТРАДИКТОРНОСТЬ) Отношение между понятиями, одно из которых выражает наличие каких-либо признаков, а другое – их отсутствие, то есть просто отрицает эти признаки, не заменяя их никакими другими. 1) А – высокий дом В – невысокий дом 2) А – выигрышный билет В – невыигрышный билет Т.е. понятия А и не-А исчерпывают весь объём понятия, в которое они входят, так как между ними нельзя поставить никакое дополнительное понятие.

Упражнение : Определите вид отношений по объёму приведённых ниже понятий. Изобразите их с помощью кругов Эйлера .


1) А – горячий чай; В – холодный чай; С – чай с лимоном

Горячий чай (В) и холодный чай (С) – находятся в отношении противоположности.

Чай с лимоном (С) может быть как горячим,

так и холодным, но может быть и, например, тёплым.

2) А – деревянный; В – каменный; С – строение; D – дом.

Всякое ли строение (С) – дом (D)? – Нет.

Всякий ли дом (D) – строение (С)? – Да.

Что-то деревянное (А) обязательно ли дом (D) или строение (С) – Нет.

Но можно найти деревянное строение (например, будка),

также можно найти деревянный дом.

Что-то каменное (В) не обязательно дом (D) или строение (С).

Но может быть и каменное строение, и каменный дом.

3) А – российский город; В – столица России;

С – Москва; D – город на Волге; Е – Углич.

Столица России (В) и Москва (С) – один и тот же город.

Углич (Е) является городом на Волге (D).

При этом, Москва, Углич, как и любой город на Волге,

являются российскими городами (А)

Задача 1 .

Каждый из 35 шестиклассников является читателем, по крайней мере, одной из двух библиотек: школьной и районной. Из них 25 человек берут книги в школьной библиотеке, 20 – в районной.

Сколько шестиклассников:

1. Являются читателями обеих библиотек;
2. Не являются читателями районной библиотеки;
3. Не являются читателями школьной библиотеки;
4. Являются читателями только районной библиотеки;
5. Являются читателями только школьной библиотеки?

Заметим, что первый вопрос является ключевым для понимания и решения данной задачи. Ведь не сразу сообразишь, как получается 20 + 25 = 45 из 35. В первом вопросе звучит подсказка к пониманию условия: есть ученики, которые посещают обе библиотеки. А если условие задачи изобразить на схеме, то ответ на первый вопрос становится очевидным.

Решение.

1. 20 + 25 – 35 = 10 (человек) – являются читателями обеих библиотек. На схеме это общая часть кругов. Мы определили единственную неизвестную нам величину. Теперь, глядя на схему, легко даем ответы на поставленные вопросы.

2. 35 – 20 = 15 (человек) – не являются читателями районной библиотеки. (На схеме левая часть левого круга)

3. 35 – 25 = 10 (человек) – не являются читателями школьной библиотеки. (На схеме правая часть правого круга)

4. 35 – 25 = 10 (человек) – являются читателями только районной библиотеки. (На схеме правая часть правого круга)

5. 35 – 20 = 15 (человек) – являются читателями только школьной библиотеки. (На схеме левая часть левого круга).

Очевидно, что 2 и 5 , а также 3 и 4 – равнозначны и ответы на них совпадают .

При решении данной задачи мы использовали способ ее графического представления при помощи так называемых кругов Эйлера. Этот способ был предложен Леонардом Эйлером и широко используется при решении логических задач.

Леона́рд Э́йлер (4(15) апреля 1707, Базель, Швейцария – 7(18) сентября 1783, Санкт-Петербург, Российская империя) – швейцарский, немецкий и российский математик, внёсший значительный вклад в развитие математики, а также механики, физики, астрономии и ряда прикладных наук. Некоторые из его потомков до сих пор живут в России.

Рассмотрим еще один пример.

Задача 2.

Часть жителей нашего дома выписывают только газету «Комсомольская правда», часть – только газету «Известия», а часть – и ту, и другую газету. Сколько процентов жителей дома выписывают обе газеты, если на газету «Комсомольская правда» из них подписаны 85%, а на «Известия» – 75%?

Решение.

Здесь нет принципиального отличия от решения предыдущей. На готовом рисунке заменим данные: 25 на 85% и 20 на 75%. Учитывая, что все жители дома составляют 100%, заменяем 35 на 100% и получаем готовое решение: 85% + 75% – 100% = 60%.

Ответ: обе газеты выписывают 60% жителей.

Чем более сложная и запутанная логическая задача, связанная с множествами, тем более очевиден эффект от применения кругов Эйлера. Только после составления рисунка их решение становится достаточно очевидным.

Задача 3.

В трёх седьмых классах 70 ребят. Из них 27 занимаются в драмкружке, 32 поют в хоре, 22 увлекаются спортом. В драмкружке 10 ребят из хора, в хоре 6 спортсменов, в драмкружке 8 спортсменов; 3 спортсмена посещают и драмкружок и хор. Сколько ребят не поют в хоре, не увлекаются спортом и не занимаются в драмкружке? Сколько ребят заняты только спортом?

Решение.

Пусть
Д – драмкружок,
Х – хор,
С – спорт.

Тогда
в круге Д – 27 ребят,
в круге Х – 32 человека,
в круге С – 22 ученика.

Те 10 ребят из драмкружка, которые поют в хоре, окажутся в общей части кругов Д и X. Трое из них ещё и спортсмены, они окажутся в общей части всех трёх кругов. Остальные семеро спортом не увлекаются. Аналогично, 8 – 3 = 5 спортсменов, не поющих в хоре и 6 – 3 = 3, не посещающих драмкружок.

Легко видеть, что 5 + 3 + 3 = 11 спортсменов посещают хор или драмкружок,

22 – (5 + 3 + 3) = 11 занимаются только спортом;

70 – (11 + 12 + 19 + 7 + 3 + 3 + 5) = 10 – не поют в хоре, не занимаются в драмкружке, не увлекаются спортом.

Ответ: 10 человек и 11 человек.

Задача 4.

В классе 30 человек. 20 из них каждый день пользуются метро, 15 – автобусом, 23 – троллейбусом, 10 – и метро, и троллейбусом, 12 – и метро, и автобусом, 9 – и троллейбусом, и автобусом. Сколько человек ежедневно пользуется всеми тремя видами транспорта?

Решение.

1 способ. Для решения опять воспользуемся кругами Эйлера. Пусть х человек пользуется всеми тремя видами транспорта. Тогда пользуются
только метро и троллейбусом – (10 – х) человек,
только автобусом и троллейбусом – (9 – х) человек,
только метро и автобусом – (12 – х) человек.

Найдем, сколько человек пользуется одним только метро:
20 – (12 – х) – (10 – х) – х = х – 2.

Аналогично получаем: х – 6 – только автобусом и х + 4 – только троллейбусом, так как всего 30 человек, составляем уравнение:
х + (12 – х) + (9 – х) + (10 – х) + (х + 4) + (х – 2) + (х – 6) = 30,
отсюда х = 3.

2 способ. А можно эту задачу решить задачу другим способом: 20 + 15 + 23 – 10 – 12 – 9 + х = 30, 27 + х = 30, х = 3 . Здесь сложили количество учеников, которые пользуются хотя бы одним видом транспорта и из полученной суммы вычли количество тех, кто пользуется двумя или тремя видами и, поэтому, вошли в сумму 2-3 раза. Таким образом, получили количество всех учеников в классе.

Ответ. 3 человека ежедневно пользуются всеми тремя видами транспорта.

Остались вопросы? Не знаете, как решить задачу?
Чтобы получить помощь репетитора – зарегистрируйтесь .
Первый урок – бесплатно!

сайт, при полном или частичном копировании материала ссылка на первоисточник обязательна.

кругами Эйлера называют фигуры, условно изображающие множества и наглядно иллюстрирующие некоторые свойства операций над множествами. В литературе круги Эйлера иногда называют диаграммами Вен на (или диаграммами Эйлера – Венна). Круги Эйлера, иллюстрирующие основные операции над множествами, представлены на рис. 1.2 (множества, полученные в результате этих операций, отмечены штриховкой). АПВ 00 АЬВ Рис. 1.2 Пример 1.8. При помощи кругов Эйлера установим сначаг ла справедливость первого соотношения, выражающего свойство дистрибутивности операций объединения и пересечения множеств, На рис. 1.3,а вертикально заштрихован круг, изображающий множество А) а горизонтально – область, отвечающая пересечению множеств В и С. В итоге тем или иным способом заштрихована область, изображающая множество A U (БПС). На рис. 1.3,5 вертикально заштрихована область, соответствующая объединению множеств Л и Б, а горизонтально – объединению множеств Л и С, так что обоими способами заштрихована область, изображающая множество (A U В) П (A U С) и совпадающая с областью, заштрихованной каким-либо способом на рис. 1.3,а. Таким образом, круги Эйлера позволяют установить справедливость (1.10). Теперь рассмотрим второй закон де Моргана (1.7) Заштрихованная на рис. 1.4,а область изображает множество ЛИВ, а незаштрихованная часть прямоугольника Q (внешняя по отношению к заштрихованной) соответствует множеству ЛПВ. На рис. 1.4,5 части прямоугольника 12, заштрихованные вертикально и горизонтально, отвечают соответственно А и В. Тогда множеству Ли В отвечает область, заштрихованная хотя бы одним из указанных способов. Она совпадает с областью, не заштрихованной на рис. 1.4,а и отвечающей множеству ЛПБ, что устанавливает справедливость (1.11). Вопросы и задачи 1.1. Запись m|n, где m,n € Z, означает, что число m нацело делит число п (то – делитель п). Описать заданные множества при условии, что х € N: 1.2. Доказать следующие соотношения и проиллюстрировать их кругами Эйлера: . 1.3. Установить, в каком отношении (X С Y, X Э У или X = Y) находятся множества X и У, если: а Использовать для иллюстрации круги Эйлера. 1.4. Пусть Aj – множество точек, образующих стороны некоторого треугольника, вписанного в заданную окружность. Описать объединение и пересечение всех таких множеств, если треугольники: а) произвольные; б) правильные; в) прямоугольные. Найти IK и flAi ieN i en для заданных семейств множеств: 1.6. Указать, какие из представленных ниже соотношений неверны, и объяснить, почему: 1.7. Указать, какие из множеств равны между собой: . 1.8. Найти множества Ли В, АГ\В, А\В, В\А и изобразить их на числовой прямой, если А = (1.0. Считая отрезок универсальным множеством, найти и изобразить на числовой прямой дополнения множеств: . 1.10. По приведенным ниже описаниям множеств людей подберите для каждой записи высказывания на языке множеств подходящую пословицу или поговорку. Надеемся, что это позволит лишний раз проанализировать смысл народных изречений. Например, если Z -множество людей, которые сами как следует не знают того, о чем говорят, то запись х £ Z можно отнести к пословице „Слышал звон, да не знает, где он, поскольку именно так говорят о человеке, наделенном указанным свойством (в данном случае – характеристическим свойством множества Z, см. 1.1). Множества людей ft – универсальное множество всех людей, Л – добрые, 5е В – незаурядные, с большими способностями, С – глупые, D – умные, Е – поступающие по своему, не слушающие советов, F – связанные корыстными отношениями, G – много обещающие, Я – не выполняющие своих обещаний, J – злоупотребляющие своим служебным положением, К – слишком важничающие, задающиеся, L – вмешивающиеся не в свое дело, М – предприимчивые, ловкие, умеющие устраиваться, Р – берущиеся за несколько дел сразу, Q – плодотворно работающие, S – ошибающиеся, Т – чувствующие вину и возможность расплаты, U – не добивающиеся результатов, V – выдающие себя своим поведением, W- недальновидные, X – действующие заодно, не предающие друг друга, У – бывалые, опытные люди. 1 + ns, Vs>-1 (неравенство Бернулли). 1.14. Доказать, что среднее арифметическое п положительных действительных чисел не меньше их среднего геометрического, т.е. п 1.15. Брауну, Джонсу и Смиту предъявлено обвинение в соучастии в ограблении банка. Похитители скрылись на поджидавшем их автомобиле. На следствии Браун показал, что это был синий „Бьюик”, Джонс – голубой „Крайслер”, а Смит – „Форд Мустанг”, но не синий. Какого цвета был автомобиль и какой марки, если известно, что, желая запутать следствие, каждый из них указал правильно либо только марку машины, либо только ее цвет? 1.1в. Для полярной экспедиции из восьми претендентов А, В, С, Д J5, F, G и Я надо отобрать шесть специалистов: биолога, гидролога, синоптика, радиста, механика и врача. Обязанности биолога могут выполнять Е и G, гидролога – В и F, синоптика – F и G, радиста – С и Д механика – С и Я, врача – А и Д но каждый из них, если будет в экспедиции, сможет выполнять лишь одну обязанность. Кого и кем следует взять в экспедицию, если F не может ехать без D – без Я и без С, С не может ехать с G, а Д – с В?

Леонард Эйлер – величайший из математиков,написал более 850 научных работ. В одной из них и появились эти круги.

Учёный писал, что «они очень подходят для того, чтобы облегчить наши размышления».

Круги Эйлера – это геометрическая схема, которая помогает находить и/или делать более наглядными логические связи между явлениями и понятиями. А также помогает изобразить отношения между каким-либо множеством и его частью.

Задача 1

Из 90 туристов, отправляющихся в путешествие, немецким языком владеют 30 человек, английским – 28 чел, французским – 42 чел. Английским и немецким одновременно владеют 8 человек, английским и французским -10 чел, немецким и французским – 5 чел, всеми тремя языками – 3 чел. Сколько туристов не владеют ни одним языком?

Решение:

Покажем условие задачи графически – с помощью трёх кругов

Ответ: 10 человек.

Задача 2

Многие ребята нашего класса любят футбол, баскетбол и волейбол. А некоторые – даже два или три из этих видов спорта. Известно, что 6 человек из класса играют только в волейбол, 2 – только в футбол, 5 – только в баскетбол. Только в волейбол и футбол умеют играть 3 человека, в футбол и баскетбол – 4, в волейбол и баскетбол – 2. Один человек из класса умеет играть во все игры, 7 не умеют играть ни в одну игру. Требуется найти:

Сколько всего человек в классе?

Сколько человек умеют играть в футбол?

Сколько человек умеют играть в волейбол?


Задача 3

В детском лагере отдыхало 70 ребят. Из них 20 занимаются в драмкружке, 32 поют в хоре, 22 увлекаются спортом. В драмкружке 10 ребят из хора, в хоре 6 спортсменов, в драмкружке 8 спортсменов, а 3 спортсмена посещают и драмкружок, и хор. Сколько ребят не поют в хоре, не увлекаются спортом и не занимаются в драмкружке? Сколько ребят заняты только спортом?

Задача 4

Из сотрудников фирмы 16 побывали во Франции, 10 – в Италии, 6 – в Англии. В Англии и Италии – пятеро, в Англии и Франции – 6, во всех трёх странах – 5 сотрудников. Сколько человек посетили и Италию, и Францию, если всего в фирме работает 19 человек, и каждый их них побывал хотя бы в одной из названных стран?

Задача 5

Шестиклассники заполняли анкету с вопросами об их любимых мультфильмах. Оказалось, что большинству из них нравятся «Белоснежка и семь гномов», «Губка Боб Квадратные Штаны» и «Волк и теленок». В классе 38 учеников. «Белоснежка и семь гномов» нравится 21 ученику. Причем трем среди них нравятся еще и «Волк и теленок», шестерым – «Губка Боб Квадратные Штаны», а один ребенок одинаково любит все три мультфильма. У «Волка и теленка» 13 фанатов, пятеро из которых назвали в анкете два мультфильма. Надо определить, скольким же шестиклассникам нравится «Губка Боб Квадратные Штаны».

Задачи для решения учащимися

1. В классе 35 учеников. Все они являются читателями школьной и район­ной библиотек. Из них 25 берут книги в школьной библиотеке, 20 – в рай­онной. Сколько из них:

а) не являются читателями школь­ной библиотеки;

б) не являются читателями район­ной библиотеки;

в) являются читателями только школьной библиотеки;

г) являются читателями только рай­онной библиотеки;

д) являются читателями обеих библиотек?

2.Каждый ученик в классе изучает английский или немецкий язык, или оба этих языка. Английский язык изучают 25 человек, немецкий – 27 человек, а тот и другой – 18 человек. Сколько всего учеников в классе?

3.На листе бумаги начертили круг площадью 78 см2 и квадрат площадью 55 см2. Площадь пересечения круга и квадрата равна 30 см2. Не занятая кру­гом и квадратом часть листа имеет пло­щадь 150 см2. Найдите площадь листа.

4. В группе туристов 25 человек. Среди них 20 человек моложе 30 лет и 15 человек старше 20 лет. Может ли так быть? Если может, то в каком случае?

5. В детском саду 52 ребенка. Каж­дый из них любит пирожное или моро­женое, или то и другое. Половина де­тей любит пирожное, а 20 человек – пирожное и мороженое. Сколько де­тей любит мороженое?

6. В классе 36 человек. Ученики это­го класса посещают математический, физический и химический кружки, причем математический кружок по­сещают 18 человек, физический – 14, химический – 10. Кроме того, извест­но, что 2 человека посещают все три кружка, 8 человек -.и математиче­ский, и физический, 5 – и математи­ческий, и химический, 3 – и физи­ческий, и химический кружки. Сколько учеников класса не посещают ни­какие кружки?

7. После каникул классный руково­дитель спросил, кто из ребят ходил в театр, кино или цирк. Оказалось, что из 36 учеников двое не были ни в кино, ни в театре, ни в цирке. В кино побы­вали 25 человек; в театре – 11; в цир­ке – 17; и в кино, и в театре – 6; и в кино, и в цирке – 10; и в театре, и в цирке – 4. Сколько человек побы­вали в театре, кино и цирке одновре­менно?

Решение задач ЕГЭ с помощью кругов Эйлера

Задача 1

В языке запросов поискового сервера для обозначения логической операции «ИЛИ» используется символ «|», а для логической операции «И» – символ «&».

Крейсер & Линкор ? Считается, что все вопросы выполняются практически одновременно, так что набор страниц, содержащих все искомые слова, не изменялся за время выполнения запросов.

Запрос Найдено страниц (в тысячах)
Крейсер | Линкор 7000
Крейсер 4800
Линкор 4500

Решение:

При помощи кругов Эйлера изобразим условия задачи. При этом цифры 1, 2 и 3 используем, чтобы обозначить полученные в итоге области.

Опираясь на условия задачи, составим уравнения:

  1. Крейсер | Линкор: 1 + 2 + 3 = 7000
  2. Крейсер: 1 + 2 = 4800
  3. Линкор: 2 + 3 = 4500

Чтобы найти Крейсер & Линкор (обозначенный на чертеже как область 2), подставим уравнение (2) в уравнение (1) и выясним, что:

4800 + 3 = 7000, откуда получаем 3 = 2200.

Теперь этот результат мы можем подставить в уравнение (3) и выяснить, что:

2 + 2200 = 4500, откуда 2 = 2300.

Ответ: 2300 – количество страниц, найденных по запросу Крейсер & Линкор.

Задача 2

В языке запросов поискового сервера для обозначения

В таблице приведены запросы и количество найденных по ним страниц некоторого сегмента сети Интернет.
Запрос
Найдено страниц (в тысячах)
Торты | Пироги
12000
Торты & Пироги
6500
Пироги
7700

Какое количество страниц (в тысячах) будет найдено по запросу Торты ?

Решение

Для решения задачи отобразим множества Тортов и Пирогов в виде кругов Эйлера.

А , Б , В ).

Из условия задачи следует:

Торты │Пироги = А + Б + В = 12000

Торты & Пироги = Б = 6500

Пироги = Б + В = 7700

Чтобы найти количество Тортов (Торты = А + Б ), надо найти сектор А Торты│Пироги ) отнимем множество Пироги.

Торты│Пироги – Пироги = А + Б + В -(Б + В ) = А = 1200 – 7700 = 4300

Сектор А равен 4300, следовательно

Торты = А + Б = 4300+6500 = 10800

Задача 3

|”, а для логической операции “И” – символ “&”.

В таблице приведены запросы и количество найденных по ним страниц некоторого сегмента сети Интернет.
Запрос
Найдено страниц (в тысячах)
Пироженое & Выпечка
5100
Пироженое
9700
Пироженое | Выпечка
14200

Какое количество страниц (в тысячах) будет найдено по запросуВыпечка ?

Считается, что все запросы выполнялись практически одновременно, так что набор страниц, содержащих все искомые слова, не изменялся за время выполнения запросов.

Решение

Для решения задачи отобразим множества Пироженых и Выпечек в виде кругов Эйлера.

Обозначим каждый сектор отдельной буквой (А , Б , В ).

Из условия задачи следует:

Пироженое & Выпечка = Б = 5100

Пироженое = А + Б = 9700

Пироженое │ Выпечка = А + Б + В = 14200

Чтобы найти количество Выпечки (Выпечка = Б + В ), надо найти сектор В , для этого из общего множества (Пироженое │ Выпечка) отнимем множество Пироженое .

Пироженое │ Выпечка – Пироженное = А + Б + В -(А + Б ) = В = 14200–9700 = 4500

Сектор В равен 4500, следовательноВыпечка = Б + В = 4500+5100 = 9600

Задача 4
убывания
Для обозначения логической операции “ИЛИ” используется символ ” |”, а для логической операции “И” – символ “&”.
Решение

Представим множества овчарок, терьеров и спаниелей в виде кругов Эйлера, обозначим сектора буквами (

А , Б , В , Г ).

с паниели │(терьеры & овчарки) = Г + Б

с паниели│овчарки = Г + Б + В

спаниели│терьеры│овчарки = А + Б + В + Г

терьеры & овчарки = Б

Расположим номера запросов в порядке убывания количества страниц: 3 2 1 4

Задача 5

В таблице приведены запросы к поисковому серверу. Расположите номера запросов в порядке возрастания количества страниц, которые найдет поисковый сервер по каждому запросу.
Для обозначения логической операции “ИЛИ” используется символ ” |”, а для логической операции “И” – символ “&”.

1
барокко | классицизм | ампир
2
барокко | (классицизм & ампир)
3
классицизм & ампир
4
барокко | классицизм

Решение

Представим множества классицизм, ампир и классицизм в виде кругов Эйлера, обозначим сектора буквами (А , Б , В , Г ).

Преобразим условие задачи в виде суммы секторов:

барокко│ классицизм │ампир = А + Б + В + Г
барокко │(классицизм & ампир) = Г + Б
классицизм & ампир = Б
барокко│ классицизм = Г + Б + А

Из сумм секторов мы видим какой запрос выдал больше количества страниц.

Расположим номера запросов в порядке возрастания количества страниц: 3 2 4 1

Задача 6
В таблице приведены запросы к поисковому серверу. Расположите номера запросов в порядке возрастания количества страниц, которые найдет поисковый сервер по каждому запросу.
Для обозначения логической операции “ИЛИ” используется символ ” |”, а для логической операции “И” – символ “&”.

1
канарейки | щеглы | содержание
2
канарейки & содержание
3
канарейки & щеглы & содержание
4
разведение & содержание & канарейки & щеглы

Решение

Для решения задачи представим запросы в виде кругов Эйлера.

K – канарейки,

Щ – щеглы,

Р – разведение.

канарейки | терьеры | содержание канарейки & содержание канарейки & щеглы & содержание разведение & содержание & канарейки & щеглы








Самая большая область закрашенных секторов у первого запроса, затем у второго, затем у третьего, а у четвертого запроса самый маленький.

В порядке возрастания по количеству страниц запросы будут представлены в следующем порядке: 4 3 2 1

Обратите внимание что в первом запросе закрашенные сектора кругов Эйлера содержат в себе закрашенные сектора второго запроса, а закрашенные сектора второго запроса содержат закрашенные сектора третьего запроса, закрашенные сектора третьего запроса содержат закрашенный сектор четвертого запроса.

Только при таких условиях мы можем быть уверены, что правильно решили задачу.

Задача 7 (ЕГЭ 2013)

В языке запросов поискового сервера для обозначения логической операции «ИЛИ» используется символ «|», а для логической операции «И» – символ «&».

В таблице приведены запросы и количество найденных по ним страниц некоторого сегмента сети Интернет.

ЗапросНайдено страниц
(в тысячах)
Фрегат | Эсминец3400
Фрегат & Эсминец900
Фрегат2100

Какое количество страниц (в тысячах) будет найдено по запросу Эсминец ?
Считается, что все запросы выполнялись практически одновременно, так что набор страниц, содержащих все искомые слова, не изменялся за время выполнения запросов.

Решение задач с помощью кругов Эйлера

Пояснительная записка

Очень часто решение задачи помогает найти рисунок. Использование рисунка делает решение простым и наглядным.

В данной разработке приведены примеры решения задач с помощью кругов Эйлера. Это не просто занимательная и интересная штука, но и весьма полезный метод решения задач. Они помогают быстро и просто решить даже достаточно сложные или просто запутанные на первый взгляд задачи.

С данным способом решения задач учащихся можно познакомить как на уроках, так и на кружковых занятиях.

Главной целью этой работы является помощь учителям математики для подготовки учащихся к олимпиадам, а также к экзаменам.

Основные понятия

Понятие множества − одно из первичных в математике. Поэтому очень трудно дать ему какое-либо определение, которое бы не заменяло слово «множество» каким-нибудь равнозначным выражением, например, совокупность, собрание элементов и т.д. Элементы множества − это то, из чего это множество состоит, например, каждый ученик вашего класса есть элемент множества школьников.

Пересечение множеств в теории множеств – это множество, которому принадлежат те и только те элементы, которые одновременно принадлежат всем данным множествам.

Круги Эйлера – геометрическая схема, с помощью которой можно изобразить отношения между подмножествами, для наглядного представления. Изобретены Леонардом Эйлером. Используется в математике, логике, менеджменте и других прикладных направлениях.

2. Решение задач с помощью кругов Эйлера
2.1. “Обитаемый остров” и “Стиляги”

Некоторые ребята из нашего класса любят ходить в кино. Известно, что 15 ребят смотрели фильм «Обитаемый остров», 11 человек – фильм «Стиляги», из них 6 смотрели и «Обитаемый остров», и «Стиляги». Сколько человек смотрели только фильм «Стиляги»?

Решение:

Чертим два множества таким образом:

6 человек, которые смотрели фильмы «Обитаемый остров» и «Стиляги», помещаем в пересечение множеств.

1. 15 – 6 = 9 – человек, которые смотрели только «Обитаемый остров»,

2. 11- 6 = 5 – человек, которые смотрели только «Стиляги».

Получаем:

Ответ: 5 человек.

2.2. Задача про библиотеки

Каждый из 35 шестиклассников является читателем, по крайней мере, одной из двух библиотек: школьной и районной. Из них 25 человек берут книги в школьной библиотеке, 20 – в районной.

Сколько шестиклассников:

  1. Являются читателями обеих библиотек;
  2. Не являются читателями районной библиотеки;
  3. Не являются читателями школьной библиотеки;
  4. Являются читателями только районной библиотеки;
  5. Являются читателями только школьной библиотеки?

Решение:

Чертим два множества таким образом:

1) 20+ 25 – 35 = 10 (человек) – являются читателями обеих библиотек. На схеме это общая часть кругов. Мы определили единственную неизвестную нам величину. Теперь, глядя на схему, легко даем ответы на поставленные вопросы.

2) 35 – 20 = 15 (человек) – не являются читателями районной библиотеки,

3) 35 – 25 = 10 (человек) – не являются читателями школьной библиотеки,

4) 35- 20 = 10 (человек) – являются читателями только районной библиотеки,

5) 35- 20 = 15 (человек) – являются читателями только школьной библиотеки.

Очевидно, что вопросы 2 и 5, а также 3 и 4 – равнозначны и ответы на них совпадают.

Ответ: 10 человек; 15 человек; 10 человек; 10 человек; 15 человек.

2.3. Гарри Поттер, Рон и Гермиона

На полке стояло 26 волшебных книг по заклинаниям, все они были прочитаны. Из них 4 прочитал и Гарри Поттер, и Рон. Гермиона прочитала 7 книг, которых не читали ни Гарри Поттер, ни Рон, и две книги, которые читал Гарри Поттер. Всего Гарри Поттер прочитал 11 книг. Сколько книг прочитал только Рон?

Решение:

Учитывая условия задачи, сделаем чертеж:

Так как Гарри Поттер всего прочитал 11 книг, из них 4 книги читал Рон и 2 книги – Гермиона, то 11 – 4 – 2 = 5 – книг прочитал только Гарри.

Следовательно, 26 – 7 – 2 – 5 – 4 = 8 – книг прочитал только Рон.

Ответ: 8 книг.

2.4. Задача про любимые мультфильмы

Шестиклассники заполняли анкету с вопросами об их любимых мультфильмах. Оказалось, что большинству из них нравятся «Белоснежка и семь гномов», «Губка Боб Квадратные Штаны» и «Волк и теленок». В классе 38 учеников. «Белоснежка и семь гномов» нравится 21 ученику. Причем трем среди них нравятся еще и «Волк и теленок», шестерым – «Губка Боб Квадратные Штаны», а один ребенок одинаково любит все три мультфильма. У «Волка и теленка» 13 фанатов, пятеро из которых назвали в анкете два мультфильма. Надо определить, скольким же шестиклассникам нравится «Губка Боб Квадратные Штаны».

Решение:

Чертим три круга, таким образом:

Из условия знаем, что трем ученикам нравиться и «Белоснежка и семь гномов», и «Волк и теленок», шестерым – «Белоснежка и семь гномов» и «Губка Боб Квадратные Штаны», а один ребенок одинаково любит все три мультфильма.

Мы помним, что по условиям задачи среди фанатов мультфильма «Волк и теленок» пятеро ребят выбрали два мультфильма сразу, т.е. 5 – 3 = 2 – ученика выбрали «Волк и теленок» и «Губка Боб Квадратные Штаны».

1) 21 – 3 – 1 – 6 = 11 – учеников выбрали только «Белоснежка и семь гномов»,

2) 13 – 3 – 1 – 2 = 7 – учеников выбрали – «Волк и теленок»,

3) 38 – (11 + 3 + 1 + 2 + 6 + 7) = 8 – ребят выбрали «Губка Боб Квадратные Штаны».

4) 8 + 2 + 1 + 6 = 17 – человек выбрали мультик «Губка Боб Квадратные Штаны».

Ответ: 17 учеников.

2.5. Задача про Крейсер и Линкор

В таблице приведены запросы и количество найденных по ним страниц некоторого сегмента сети интернет.

Запрос

Найдено страниц, тыс.

Крейсер и Линкор

7000

Крейсер

4800

Линкор

4500

Какое количество страниц (в тысячах) будет найдено по запросу Крейсер и Линкор? (Считается, что все вопросы выполняются практически одновременно, так что набор страниц, содержащих все искомые слова, не изменялся за время выполнения запросов.)

Решение:

При помощи кругов Эйлера изобразим условия задачи.

1) 4800 + 4500 – 7000 = 2300 (тыс. страниц) – найдено по запросу Крейсер и Линкор,

2) 4800 – 2300 = 2500 (тыс. страниц) – найдено по запросу Крейсер,

3) 4500 – 2300 = 2200 (тыс. страниц) – найдено по запросу Линкор.

Ответ: 2300 тыс. страниц.

2.6. Задача про блондинок

Каждый ученик класса – либо девочка, либо блондин, либо любит математику. В классе 20 девочек, из них 12 блондинок, но одна блондинка любит математику. Всего в классе 24 ученика – блондина, математику из них любят 12, а всего учеников (мальчиков и девочек), которые любят математику, 17, из них 6 девочек. Сколько учеников в данном классе?

Решение:

Изобразим с помощью кругов Эйлера данные из задачи:

1) 12 – 1 = 11 (учеников) – девочек блондинок,

2) 12 – 1 = 11 (учеников) – блондины и любят математику,

3) 6 – 1 = 5 (учеников) – девочек, которые любят математику,

4) 20 – 11 – 1 – 5 = 3 (ученика) – девочки,

5) 24 – 11 – 1 – 11 = 1 (ученик) – блондин,

6) 17- 5 – 1 – 11 = 0 (учеников) – любят математику,

7) 3 + 1 + 0 + 5 + 11 + 11 + 1 = 32 (ученика) – всего в классе.

Ответ: 32 ученика.

2.7. Задача про кружки

В трёх седьмых классах 70 ребят. Из них 27 занимаются в драмкружке, 32 поют в хоре, 22 увлекаются спортом. В драмкружке 10 ребят из хора, в хоре 6 спортсменов, в драмкружке 8 спортсменов; 3 спортсмена посещают и драмкружок и хор. Сколько ребят не поют в хоре, не увлекаются спортом и не занимаются в драмкружке? Сколько ребят заняты только спортом?

Решение:

Учитывая условия задачи, сделаем чертеж:

1) 10 – 3 = 7 (ребят) – посещают драмкружок и хор,

2) 6 – 3 = 3 (ребят) – поют в хоре и занимаются спортом,

3) 8 – 3 = 5 (ребят) – занимаются спортом и посещают драмкружок,

4) 27 – 7 – 3 – 5 = 12 (ребят) – посещают драмкружок,

5) 32 – 7 3 – 3 = 19 (ребят) – поют в хоре,

6) 22 – 5 – 3 – 3 = 11 (ребят) – увлекаются спортом,

7) 70 – (12 + 19 + 11 + 5+ 7 + 3 + 3) = 10 (ребят) – не поют в хоре, не увлекаются спортом и не занимаются в драмкружке.

Ответ: 10 человек и 11 человек.

Задачи для самостоятельного решения

1. На фирме работают 67 человек. Из них 47 знают английский язык, 35 – немецкий язык, а 23 – оба языка. Сколько человек фирмы не знают ни английского, ни немецкого языков?

2. Из 40 учащихся нашего класса 32 любят молоко, 21 – лимонад, а 15 – и молоко, и лимонад. Сколько ребят в нашем классе не любят ни молоко, ни лимонад?

3. 12 моих одноклассников любят читать детективы, 18 – фантастику, трое с удовольствием читают и то, и другое, а один вообще ничего не читает. Сколько учеников в нашем классе?

4. Из тех 18 моих одноклассников, которые любят смотреть триллеры, только 12 не прочь посмотреть и мультфильмы. Сколько моих одноклассников смотрят одни «мультики», если всего в нашем классе 25 учеников, каждый из которых любит смотреть или триллеры, или мультфильмы, или и то и другое?

5. Из 29 мальчишек нашего двора только двое не занимаются спортом, а остальные посещают футбольную или теннисную секции, а то и обе. Футболом занимается 17 мальчишек, а теннисом – 19. Сколько футболистов играет в теннис? Сколько теннисистов играет в футбол?

6. В одном классе 25 учеников. Из них 7 любят груши, 11 – черешню. Двое любят груши и черешню; 6 – груши и яблоки; 5 – яблоки и черешню. Но есть в классе два ученика, которые любят все и четверо таких, что не любят фруктов вообще. Сколько учеников этого класса любят яблоки?

7. В конкурсе красоты участвовали 22 девушки. Из них 10 было красивых, 12 – умных и 9 – добрых. Только 2 девушки были и красивыми, и умными; 6 девушек были умными и одновременно добрыми. Определите, сколько было красивых и в то же время добрых девушек, если я скажу вам, что среди участниц не оказалось ни одной умной, доброй и вместе с тем красивой девушки?

8. В нашем классе 35 учеников. За первую четверть пятерки по русскому языку имели 14 учеников; по математике – 12; по истории – 23. По русскому и математике – 4; по математике и истории – 9; по русскому языку и истории – 5. Сколько учеников имеют пятерки по всем трем предметам, если в классе нет ни одного ученика, не имеющего пятерки хотя бы по одному из этих предметов?

9. Из 100 человек 85 знают английский язык, 80 – испанский, 75 – немецкий. Все владеют, по крайней мере, одним иностранным языком. Среди них нет таких, которые знают два иностранных языка, но есть владеющие тремя языками. Сколько человек из этих 100 знают три языка?

10. Из сотрудников фирмы 16 побывали во Франции, 10 – в Италии, 6 – в Англии; в Англии и Италии – 5; в Англии и Франции – 6; во всех трех странах – 5 сотрудников. Сколько человек посетили и Италию, и Францию, если всего в фирме работают 19 человек, и каждый из них побывал хотя бы в одной из названных стран?

Список использованных источников

1. Баженов И.И, Порошкин А.Г., Тимофеев А.Ю., Яковлев В.Д. Задачи для школьных математических кружков: учеб. пособие / Сыктывкар: Сыктывкарский университет, 2006.

2. Марков И.С. Новые олимпиады по математике – Ростов н/Д: Феникс, 2005.

3. https://ru.wikipedia.org/wiki/

4. http://logika.vobrazovanie.ru

5. http://www.otvet-prost.ru/load/diskretnaja_matematika/na_krugi_ehjlera/zadacha_na_krugi_ehjlera/18-1-0-22

6. http://urok.1sept.ru/articles/550092/

7. http://www.tutoronline.ru/blog/reshit-zadachu-pomogut-krugi-jejlera

Задачи на пересечение и объединение множеств (Круги Эйлера)

Круги Эйлера – задачи на пересечение или объединение множеств


Это новый тип задач, в которых требуется найти некоторое пересечение множеств или их объединение, соблюдая условия задачи.
Круги Эйлера — геометрическая схема, с помощью которой можно изобразить отношения между подмножествами, для наглядного представления.
Метод Эйлера является незаменимым при решении некоторых задач, а также упрощает рассуждения. Однако, прежде чем приступить к решению задачи, нужно проанализировать условие. Иногда с помощью арифметических действий решить задачу легче.

“Обитаемый остров” и “Стиляги”

Некоторые ребята из нашего класса любят ходить в кино. Известно, что 15 ребят смотрели фильм «Обитаемый остров», 11 человек – фильм «Стиляги», из них 6 смотрели и «Обитаемый остров», и «Стиляги». Сколько человек смотрели только фильм «Стиляги»?

Решение

Чертим два множества таким образом:


6 человек, которые смотрели фильмы «Обитаемый остров» и «Стиляги», помещаем в пересечение множеств.
15 – 6 = 9 – человек, которые смотрели только «Обитаемый остров».
11 – 6 = 5 – человек, которые смотрели только «Стиляги».
Получаем:


Ответ. 5 человек смотрели только «Стиляги».

Любимые мультфильмы

Среди школьников шестого класса проводилось анкетирование по любимым мультфильмам. Самыми популярными оказались три мультфильма: «Белоснежка и семь гномов», «Губка Боб Квадратные Штаны», «Волк и теленок». Всего в классе 38 человек. «Белоснежку и семь гномов» выбрали 21 ученик, среди которых трое назвали еще «Волк и теленок», шестеро – «Губка Боб Квадратные Штаны», а один написал все три мультфильма. Мультфильм «Волк и теленок» назвали 13 ребят, среди которых пятеро выбрали сразу два мультфильма. Сколько человек выбрали мультфильм «Губка Боб Квадратные Штаны»?

Решение

В этой задаче 3 множества, из условий задачи видно, что все они пересекаются между собой. Получаем такой чертеж:


Учитывая условие, что среди ребят, которые назвали мультфильм «Волк и теленок» пятеро выбрали сразу два мультфильма, получаем:


21 – 3 – 6 – 1 = 11 – ребят выбрали только «Белоснежку и семь гномов».
13 – 3 – 1 – 2 = 7 – ребят смотрят только «Волк и теленок».
Получаем:


38 – (11 + 3 + 1 + 6 + 2 + 7) = 8 – человек смотрят только «Губка Боб Квадратные Штаны».
Делаем вывод, что «Губка Боб Квадратные Штаны» выбрали 8 + 2 + 1 + 6 = 17 человек.
Ответ. 17 человек выбрали мультфильм «Губка Боб Квадратные Штаны».

«Мир музыки»

В магазин «Мир музыки» пришло 35 покупателей. Из них 20 человек купили новый диск певицы Максим, 11 – диск Земфиры, 10 человек не купили ни одного диска. Сколько человек купили диски и Максим, и Земфиры?

Решение

Изобразим эти множества на кругах Эйлера.


Теперь посчитаем: Всего внутри большого круга 35 покупателей, внутри двух меньших 35–10=25 покупателей. По условию задачи 20 покупателей купили новый диск певицы Максим, следовательно, 25 – 20 = 5 покупателей купили только диск Земфиры. А в задаче сказано, что 11 покупателей купили диск Земфиры, значит 11 – 5 = 6 покупателей купили диски и Максим, и Земфиры:


Ответ: 6 покупателей купили диски и Максим, и Земфиры.

Гарри Поттер, Рон и Гермиона

На полке стояло 26 волшебных книг по заклинаниям, все они были прочитаны. Из них 4 прочитал и Гарри Поттер, и Рон. Гермиона прочитала 7 книг, которых не читали ни Гарри Поттер, ни Рон, и две книги, которые читал Гарри Поттер. Всего Гарри Поттер прочитал 11 книг. Сколько книг прочитал только Рон?

Решение

Учитывая условия задачи, чертеж будет таков:


Так как Гарри Поттер всего прочитал 11 книг, из них 4 книги читал Рон и 2 книги – Гермиона, то 11 – 4 – 2 = 5 – книг прочитал только Гарри. Следовательно,
26 – 7 – 2 – 5 – 4 = 8 – книг прочитал только Рон.
Ответ. 8 книг прочитал только Рон.

Пионерский лагерь

В пионерском лагере 70 ребят. Из них 27 занимаются в драмкружке, 32 поют в хоре, 22 увлекаются спортом. В драмкружке 10 ребят из хора, в хоре 6 спортсменов, в драмкружке 8 спортсменов; 3 спортсмена посещают и драмкружок и хор. Сколько ребят не поют, не увлекаются спортом, не занимаются в драмкружке? Сколько ребят заняты только спортом?

Решение

Изобразим множества следующим образом:


70 – (6 + 8 + 10 + 3 + 13 + 6 + 5) = 19 – ребят не поют, не увлекаются спортом, не занимаются в драмкружке. Только спортом заняты 5 человек.
Ответ. 5 человек заняты только спортом.

Экстрим

Из 100 ребят, отправляющихся в детский оздоровительный лагерь, кататься на сноуборде умеют 30 ребят, на скейтборде – 28, на роликах – 42. На скейтборде и на сноуборде умеют кататься 8 ребят, на скейтборде и на роликах – 10, на сноуборде и на роликах – 5, а на всех трех – 3. Сколько ребят не умеют кататься ни на сноуборде, ни на скейтборде, ни на роликах?

Решение


Всеми тремя спортивными снарядами владеют три человека, значит, в общей части кругов вписываем число 3. На скейтборде и на роликах умеют кататься 10 человек, а 3 из них катаются еще и на сноуборде. Следовательно, кататься только на скейтборде и на роликах умеют 10-3=7 ребят. Аналогично получаем, что только на скейтборде и на сноуборде умеют кататься 8-3=5 ребят, а только на сноуборде и на роликах 5-3=2 человека. Внесем эти данные в соответствующие части. Определим теперь, сколько человек умеют кататься только на одном спортивном снаряде. Кататься на сноуборде умеют 30 человек, но 5+3+2=10 из них владеют и другими снарядами, следовательно, только на сноуборде умеют кататься 20 ребят. Аналогично получаем, что только на скейтборде умеют кататься 13 ребят, а только на роликах – 30 ребят. По условию задачи всего 100 ребят. 20+13+30+5+7+2+3=80 – ребят умеют кататься хотя бы на одном спортивном снаряде. Следовательно, 20 человек не умеют кататься ни на одном спортивном снаряде.
Ответ. 20 человек не умеют кататься ни на одном спортивном снаряде.

“Обитаемый остров” и “Стиляги”

Некоторые ребята из нашего класса любят ходить в кино. Известно, что 15 ребят смотрели фильм «Обитаемый остров», 11 человек – фильм «Стиляги», из них 6 смотрели и «Обитаемый остров», и «Стиляги». Сколько человек смотрели только фильм «Стиляги»?

Любимые мультфильмы

Среди школьников шестого класса проводилось анкетирование по любимым мультфильмам. Самыми популярными оказались три мультфильма: «Белоснежка и семь гномов», «Губка Боб Квадратные Штаны», «Волк и теленок». Всего в классе 38 человек. «Белоснежку и семь гномов» выбрали 21 ученик, среди которых трое назвали еще «Волк и теленок», шестеро – «Губка Боб Квадратные Штаны», а один написал все три мультфильма. Мультфильм «Волк и теленок» назвали 13 ребят, среди которых пятеро выбрали сразу два мультфильма. Сколько человек выбрали мультфильм «Губка Боб Квадратные Штаны»?

«Мир музыки»

В магазин «Мир музыки» пришло 35 покупателей. Из них 20 человек купили новый диск певицы Максим, 11 – диск Земфиры, 10 человек не купили ни одного диска. Сколько человек купили диски и Максим, и Земфиры?

Пионерский лагерь

В пионерском лагере 70 ребят. Из них 27 занимаются в драмкружке, 32 поют в хоре, 22 увлекаются спортом. В драмкружке 10 ребят из хора, в хоре 6 спортсменов, в драмкружке 8 спортсменов; 3 спортсмена посещают и драмкружок и хор. Сколько ребят не поют, не увлекаются спортом, не занимаются в драмкружке? Сколько ребят заняты только спортом?

Экстрим

Из 100 ребят, отправляющихся в детский оздоровительный лагерь, кататься на сноуборде умеют 30 ребят, на скейтборде – 28, на роликах – 42. На скейтборде и на сноуборде умеют кататься 8 ребят, на скейтборде и на роликах – 10, на сноуборде и на роликах – 5, а на всех трех – 3. Сколько ребят не умеют кататься ни на сноуборде, ни на скейтборде, ни на роликах?

круги Эйлера – Основы логики и логические основы компьютера

В языке запросов поискового сервера для обозначения логической операции «ИЛИ» используется символ «|», а для логической операции «И» – символ «&».

В таблице приведены запросы и количество найденных по ним страниц некоторого сегмента сети интернет.Какое количество страниц (в тысячах) будет найдено по запросу Крейсер & Линкор?Считается, что все вопросы выполняются практически одновременно, так что набор страниц, содержащих все искомые слова, не изменялся за время выполнения запросов.

При помощи кругов Эйлера изобразим условия задачи. При этом цифры 1, 2 и 3 используем, чтобы обозначить полученные в итоге области.

Опираясь на условия задачи, составим уравнения:

Чтобы найти Крейсер & Линкор (обозначенный на чертеже как область 2), подставим уравнение (2) в уравнение (1) и выясним, что:

4800 + 3 = 7000, откуда получаем 3 = 2200.

Теперь этот результат мы можем подставить в уравнение (3) и выяснить, что:

2 + 2200 = 4500, откуда 2 = 2300.

Ответ: 2300 – количество страниц, найденных по запросу Крейсер & Линкор.

В языке запросов поискового сервера для обозначения логической операции “ИЛИ” используется символ “|”, а для логической операции “И” – символ “&”.

Считается, что все запросы выполнялись практически одновременно, так что набор страниц, содержащих все искомые слова, не изменялся за время выполнения запросов.

Решение

Для решения задачи отобразим множества Тортов и Пирогов в виде кругов Эйлера.

Обозначим каждый сектор отдельной буквой (А, Б, В).

Из условия задачи следует:

Торты │Пироги =  А+Б+В = 12000

Торты & Пироги = Б = 6500

Пироги = Б+В = 7700

Чтобы найти количество Тортов (Торты = А+Б), надо найти сектор А, для этого из общего множества (Торты│Пироги) отнимем множество Пироги.

Торты│Пироги – Пироги = А+Б+В-(Б+В) = А = 1200 – 7700 = 4300

Сектор А равен 4300, следовательно

Торты = А+Б = 4300+6500 = 10800


Задача 3

В языке запросов поискового сервера для обозначения логической операции “ИЛИ” используется символ “|”, а для логической операции “И” – символ “&”.

В таблице приведены запросы и количество найденных по ним страниц некоторого сегмента сети Интернет.
Запрос
Найдено страниц (в тысячах)
Пироженое & Выпечка
5100
Пироженое
9700
Пироженое | Выпечка
14200

Какое количество страниц (в тысячах) будет найдено по запросуВыпечка?

Считается, что все запросы выполнялись практически одновременно, так что набор страниц, содержащих все искомые слова, не изменялся за время выполнения запросов.Решение 

Для решения задачи отобразим множестваПироженых и Выпечек в виде кругов Эйлера.

Обозначим каждый сектор отдельной буквой (А, Б, В).

Из условия задачи следует:

Пироженое & Выпечка = Б = 5100

Пироженое = А+Б = 9700

Пироженое │ Выпечка =  А+Б+В = 14200

Чтобы найти количество Выпечки (Выпечка = Б+В), надо найти секторВ, для этого из общего множества (Пироженое │ Выпечка ) отнимем множество Пироженое.

Пироженое │ Выпечка – Пироженное = А+Б+В-(А+Б) = В = 14200–9700 = 4500

Сектор В равен 4500, следовательно  Выпечка = Б + В = 4500+5100 =9600

Задача 4
В таблице приведены запросы к поисковому серверу. Расположите номера запросов в порядке убывания количества страниц, которые найдет поисковый сервер по каждому запросу.
Для обозначения логической операции “ИЛИ” используется символ “|”, а для логической операции “И” – символ “&”.
1
спаниели | (терьеры & овчарки)
2
спаниели | овчарки
3
спаниели | терьеры | овчарки
4
терьеры & овчарки

Решение 

Представим множества овчарок, терьеров и спаниелей в виде кругов Эйлера, обозначим сектора буквами (А, Б, В, Г).

Преобразим условие задачи в виде суммы секторов:

спаниели │(терьеры & овчарки) = Г + Б

спаниели│овчарки = Г + Б + В

спаниели│терьеры│овчарки = А + Б + В + Г

терьеры & овчарки = Б

Из сумм секторов мы видим какой запрос выдал больше количества страниц.

Расположим номера запросов в порядке убывания количества страниц: 3 2 1 4


Задача 5

В таблице приведены запросы к поисковому серверу. Расположите номера запросов в порядке возрастания количества страниц, которые найдет поисковый сервер по каждому запросу.
Для обозначения логической операции “ИЛИ” используется символ “|”, а для логической операции “И” – символ “&”.

1
барокко | классицизм | ампир
2
барокко | (классицизм & ампир)
3
классицизм & ампир
4
барокко | классицизм

Решение 

Представим множества классицизм, ампир и классицизм в виде кругов Эйлера, обозначим сектора буквами (А, Б, В, Г).

Преобразим условие задачи в виде суммы секторов:

барокко│ классицизм │ампир = А + Б + В + Г
барокко │(классицизм & ампир) = Г + Б
классицизм & ампир = Б
барокко│ классицизм = Г + Б + А

Из сумм секторов мы видим какой запрос выдал больше количества страниц.

Расположим номера запросов в порядке возрастания количества страниц: 3 2 4 1




Задача 6
В таблице приведены запросы к поисковому серверу. Расположите номера запросов в порядке возрастания количества страниц, которые найдет поисковый сервер по каждому запросу.
Для обозначения логической операции “ИЛИ” используется символ “|”, а для логической операции “И” – символ “&”.
1
канарейки | щеглы | содержание
2
канарейки & содержание
3
канарейки & щеглы & содержание
4
разведение & содержание & канарейки & щеглы

Решение 

Для решения задачи представим запросы в виде кругов Эйлера.

K –  канарейки,

Щ – щеглы,

С – содержание,

Р – разведение.

Далее будем закрашивать красным цветом сектора согласно запросам, наибольший по величине сектор даст большее количество страниц на запрос.

канарейки | терьеры | содержаниеканарейки & содержаниеканарейки & щеглы & содержаниеразведение & содержание & канарейки & щеглы




Самая большая область закрашенных секторов у первого запроса, затем у второго, затем у третьего, а у четвертого запроса самый маленький.

В порядке возрастания по количеству страниц запросы будут представлены в следующем порядке: 4 3 2 1

Обратите внимание что в первом запросе закрашенные сектора кругов Эйлера содержат в себе закрашенные сектора второго запроса, а закрашенные сектора второго запроса содержат закрашенные сектора третьего запроса, закрашенные сектора третьего запроса содержат закрашенный сектор четвертого запроса.

Только при таких условиях мы можем быть уверены, что правильно решили задачу.

 

Задача 7 (ЕГЭ 2013)

 В языке запросов поискового сервера для обозначения логической операции «ИЛИ» используется символ «|», а для логической операции «И» – символ «&». 

В таблице приведены запросы и количество найденных по ним страниц некоторого сегмента сети Интернет. 
ЗапросНайдено страниц
(в тысячах)
Фрегат | Эсминец3400
Фрегат & Эсминец900
Фрегат2100

Какое количество страниц (в тысячах) будет найдено по запросу Эсминец
Считается, что все запросы выполнялись практически одновременно, так что набор страниц, содержащих все искомые слова, не изменялся за время выполнения запросов.

Ответ: 2200

Решение: Запрос “Фрегат” обозначим символом “Ф”, “Эсминец” – символом “Э”.

Э=(Ф|Э)-Ф+(Ф&Э)=3400-2100+900=2200.








Разбор задачи B12 (демо ЕГЭ 2012)

Время выполнения-2 мин, уровень сложности-повышенный

В языке запросов поискового сервера для обозначения логической операции «ИЛИ» используется символ «|», а для логической операции «И» – символ «&».
В таблице приведены запросы и количество найденных по ним страниц некоторого сегмента сети Интернет.

ЗапросНайдено страниц
(в тысячах)
Шахматы | Теннис7770
Теннис5500
Шахматы & Теннис1000

Какое количество страниц (в тысячах) будет найдено по запросу Шахматы?
Считается, что все запросы выполнялись практически одновременно, так что набор страниц, содержащих все искомые слова, не изменялся за время выполнения запросов.

Ответ: 3270

Решение: Изобразим запросы в виде диаграмм Эйлера-Венна.

Запрос “Шахматы” обозначим символом “Ш”, “Теннис” – символом “Т”.

Ш=(Ш|Т)-Т+(Ш&Т)=7770-5500+1000=3270.


Задачи для самостоятельного решения

Задача 1

В таблице приведены запросы к поисковому серверу. Расположите номера запросов в порядке возрастания количества страниц, которые найдет поисковый сервер по каждому запросу.
Для обозначения логической операции “ИЛИ” используется символ “|”, а для логической операции “И” – символ “&”.

1
принтеры & сканеры & продажа
2
принтеры  & продажа
3
принтеры | продажа
4
принтеры | сканеры | продажа

Задача 2

В таблице приведены запросы к поисковому серверу. Расположите номера запросов в порядке возрастания количества страниц, которые найдет поисковый сервер по каждому запросу.
Для обозначения логической операции “ИЛИ” используется символ “|”, а для логической операции “И” – символ “&”.

1
физкультура
2
физкультура & подтягивания & отжимания
3
физкультура & подтягивания
4
физкультура | фитнесс


Решение логических задач с помощью кругов эйлера

Муниципальное общеобразовательное учреждение

лицей № 8 «Олимпия»

Дзержинского района г. Волгограда

Телефоны (8442) 58-80-83, 51-81-31 адрес электронной почты lyceum8@mail.ru

Решение логических задач с помощью кругов Эйлера

Выполнил:

Назаретян Сюзана Горовна,

ученица 5 Б класса

Учитель:

Кокиева Лилия Диляверовна, учитель

математики высшей категории

Волгоград, 2011

Оглавление

С.

Введение……………………………………………………………………………………

3 — 4

Глава I. Логические задачи и круги Эйлера ……………..…….……

5 — 9

1.1. Трудно решать логические задачи? …..…………………….

5 — 6

1.2. Немного о множествах ………..……………………………

6 — 8

1.3. Из истории кругов Эйлера …….……..…………………….

8 — 9

Глава II. Решение логических задач с помощью кругов Эйлера…..

7 — 14

2.1. Задачи на пересечение и объединение двух множеств…….

9 —12

2.2. Задачи на пересечение и объединение трёх множеств ……

12 — 14

Заключение………………………………………………………………………………..

15

Список источников и литературы……………………………………………….

16

Приложения ………………………………………………………………………………

17—20

Введение.

Сколько гостей Вам встречать, если собираются друзья с 15 угощениями и 20 украшениями? Может ли хватить всем места за столом, вмещающем 22 человека? Первое, что приходит на ум, это 35 человек. А причём здесь 22 человека? Есть подвох? Конечно! Ведь надо рассмотреть несколько вариантов.

Как узнать количество учащихся класса, посещающих одновременно две или три секции, если известны количества участников каждой секции отдельно? Можно ли научиться решать такие задачи, планируя результат? Хочется ответить положительно.

А как решить такую задачу: «Министерство послало в один из лицеев инспектора для проверки, как в нём ведётся преподавание иностранных языков. Сотрудник министерства в отчёте записал, что в лицее учатся 100 детей. Каждый изучает по крайней мере один из трёх языков: французский, немецкий и испанский. Причём все три языка изучают 5 человек; немецкий и испанский 10;французский и испанский 8; немецкий и французский 20; испанский 30, немецкий 23, французский 50. Инспектор, представивший отчёт, был уволен. Почему?»? Такое длинное условие: пока дочитали до конца – забыли начало. Что делать?

Оказывается, такие задачи решаются с помощью кругов Эйлера. Изображение условий задачи в виде кругов Эйлера, как правило, упрощает и облегчает путь к её решению.

Актуальность нашей работы заключается в том, чтобы такие задачи не ставили нас «в тупик» и мы могли их решать.

С учетом этого и была выбрана тема иссле­дования: «Решение логических задач с помощью кругов Эйлера».

Объект исследования — логические задачи.

Предмет исследования —использование кругов Эйлера для решения логических задач .

Гипотеза исследования. Можно решать логические задачи определённого вида специальными способами и в 5 – 6 классах.

Целью нашего исследования является исследование механизма решения определённых логических задач при помощи кругов Эйлера.

Для достижения цели исследования и обоснования гипотезы нам необходимо решить ряд задач:

  1. Найти необходимые сведения о пересечении и объединении множеств, о кругах Эйлера.

  2. Рассмотреть способы решения логических задач на пересечение и объединение двух и трёх множеств.

  3. Вывести в общем виде способ решения логических задач определённого вида с помощью кругов Эйлера.

  4. Научиться решать конкретные логические задачи с помощью кругов Эйлера.

  5. Создать модели «Круги Эйлера» для решения задач с двумя и тремя множествами в помощь учащимся.

Методы исследования:

1. Поиск, анализ и синтез различных источников информации.

2. Интервьюирование, беседы.

Практическая значимость заключается в расширении аппарата для решения логических задач. Данный материал можно будет использовать на некоторых уроках, для проведения кружков, факультативных занятий по математике. Применение кругов Эйлера придает задачам наглядность и простоту.

Теоретическая значимость заключается в разработке способа действий при решении логических задач с помощью кругов Эйлера в общем виде.

Здесь будет выводиться история переписки.

Глава I. Логические задачи и круги Эйлера

1.1. Трудно решать логические задачи?

Логика – это искусство рассуждать, умение делать правильные выводы. Это не всегда легко, потому что очень часто необходимая информация «замаскирована», представлена неявно, и надо уметь её извлечь.

Решение логических задач – одно из важнейших средств развития мыслительных способностей.

Логические задачи обладают рядом достоинств, позволяющих использовать их для развития соображения и улучшения логического мышления детей, начиная с детского сада и заканчивая старшими классами средней школы. Логические задачи допускают изложение в занимательной, игровой форме. С другой стороны, такие задачи труднее, для их решения часто не требуется глубоких знаний, а следует применить смекалку.

Вдоль овражка
Шла фуражка,
Две косынки,
Три корзинки
И от них не отставала
Белоснежная панама.
Посчитай поскорей
Сколько было детей?

Задача предполагает несколько решений. Потому что мы точно не знаем, носил ли кто – нибудь и головной убор, и корзинку.
1 Решение. Предполагается, что каждый ребёнок носил 1 предмет. Значит, детей было 7.
2 Решение. Предполагается, что 1 из детей нёс корзинку и головной убор. Следовательно, детей было 6.
3 Решение. Предполагается, что 2 из детей носили и корзинку, и головной убор. Следовательно, детей было 5 .
4 Решение. Предполагается, что 3 из детей носили и корзинку, и головной убор. Следовательно, детей было 4.

1.2. Немного о множествах

Множество – одно из основных понятий математики. Его смысл выражается словами: совокупность, собрание, класс, набор, команда и т.д. Этот смысл поясняется многочисленными примерами. Так, можно говорить о множестве всех учащихся 5-го класса, о множестве всех жителей Волгограда, о множестве всех натуральных чисел, о множестве корней данного уравнения. Основатель теории множеств немецкий математик Георг Кантор (1845–1918) так определил множество – «многое, мыслимое как единое, целое».

Множества обозначаются прописными буквами латинского алфавита А, В, С, …

О предметах, составляющих множество, говорят, что они принадлежат этому множеству или являются его элементами. Множества, элементами которых являются числа, называются числовыми множествами.

Множество может быть задано перечислением всех его элементов в произвольном порядке. Такое множество называют конечным. Мы будем рассматривать только конечные множества.

Множество, в котором нуль элементов, называют пустым.

Над множествами, как и над числами, производят операции. Рассмотрим некоторые из них: пересече­ние, объединение и разность.

Пересечение множеств

Возьмем множество X, состоящее из букв а, б, в, г, д, и множество Y, состоящее из букв г, д, е, ж:

X = {а, б, в, г, д}, Y= {г, д, е, ж}.

Эти множества имеют общие элементы гид. Множества X и Y называются пересекающимися множествами. Множество общих элементов X и Y на­зывают пересечением множеств X и Y и обозначают с помощью знака :Х Y={г, д} (рис. 1).

Пусть множество А = {1, 3, 5}. Множества А и X не имеют ни одного общего элемента. В таком случае множества А и X называются непересекающимися множествами. Пересечением множеств А и X являет­ся пустое множество: А  Х=  (рис. 2).

Пересечением множеств называется новое множество, состоящее
из элементов, принадлежащих одновременно нескольким множествам


Рис. 1

Рис. 2

Объединение множеств

Если из элементов множеств X и Y составить новое множество, состоящее из всех элементов этих мно­жеств и не содержащее других элементов, то полу­чится объединение множеств Х и Y, которое обозна­чают с помощью знака :

X и Y= {а, б, в, г, д, е, ж) (рис. 4).

Объединение множеств А и X не является пустым:

А X = {1, 3, 5, а, б, в, г, д) (рис. 5).

Объединением множеств называется новое множество, состоящее
из элементов, принадлежащих хотя бы одному из множеств.


Рис. 3

Рис. 4

Рис. 4

Разность

Разность множеств X и Y — это множество всех элементов из X, не являющихся элементами из Y.Разность обозначают Х\Y = {а, б, в} (рис. 5).


Рис. 5

1.3. Из истории кругов Эйлера

Часто множество изображают кругами, эти круги обычно называют «кругами Эйлера» по имени величайшего математика Леонарда Эйлера.

Леонард Эйлер (Euler) (1707 – 1783 г.г.) – математик, механик, физик и астроном. По происхождению швейцарец, а работал в основном в Росси и в Германии. В 1726 году был приглашен в Петербургскую АН и в 1727 году переехал в Россию. В 1741 – 1766 годах работал в Берлине, член Берлинской АН. Эйлер – ученый необычайной широты интересов и творческой продуктивности. Автор свыше 800 работ по математическому анализу, дифференциальной геометрии, теории чисел, приближенным вычислениям, небесной механике, математической физике, оптике, баллистике, кораблестроению, теории музыки и др., оказавших значительное влияние на развитие науки.

Одним из первых, кто разрабатывал метод решения задач с помощью кругов Эйлера, был выдающийся немецкий математик и философ Готфрид Вильгельм Лейбниц (1646 – 1716). В его черновых набросках были обнаружены рисунки с такими кругами. Затем этот метод довольно основательно развил швейцарский математик Леонард Эйлер (1707 – 1783). Он долгие годы работал в Петербургской Академии наук. К этому времени относятся его знаменитые «Письма к немецкой принцессе», написанные в период с 1761 по 1768 год. В некоторых из этих «Писем…» Эйлер как раз и рассказывает о кругах, которые «очень подходят для того, чтобы облегчить наши размышления». После Эйлера этот же метод разрабатывал чешский математик Бернард Больцано (1781 – 1848).

Только в отличие от Эйлера он рисовал не круговые, а прямоугольные схемы. Методом кругов пользовался и немецкий математик Эрнест Шредер (1841 – 1902). Этот метод широко используется в книге «Алгебра логики». Но наибольшего расцвета графические методы достигли в сочинениях английского логика Джона Венна (1843 – 1923). С наибольшей полнотой этот метод изложен им в книге «Символическая логика», изданной в Лондоне в 1881 году. В честь Венна вместо кругов Эйлера соответствующие рисунки называют иногда диаграммами Венна; в некоторых книгах их называют также диаграммами (или кругами) Эйлера – Венна.

Глава II. Решение логических задач с помощью кругов Эйлера

2.1. Задачи на пересечение и объединение двух множеств

К Лене на День Рождения пришли гости с подарками. Получилось так, что подарили только букеты цветов и воздушные шарики. Шесть гостей подарили букеты цветов, четыре – воздушные шарики. Сколько было гостей?
Задача предполагает несколько решений. Потому что мы точно не знаем, брал ли кто – нибудь из гостей два подарка.

1 Решение. Предполагается, что каждый гость с одним подарком. Следовательно, гостей 10.

2 Решение. Предполагается, что 1 из гостей пришел и с шариком, и с букетом цветов. Следовательно, 6 + 3 = 9 гостей.

3 Решение. Предполагается, что 2 из гостей пришли с двумя подарками. Следовательно, гостей 8.

4 Решение. Предполагается, что 3 из гостей пришли и с шариком, и с букетом цветов. Следовательно, 6 + 1 = 7.

5 Решение. Предполагается, 4 из гостей пришли с 2 подарками. Следовательно, 4 + 2 = 6 гостей.

1

Ц

) 2)

Ш

Ш

4

5

1

3

Ш

Ц

Ш

Ц

) 4)

4

2

2

3

3


Ш

Ц

5)

2

В одном множестве 40 элементов, а в другом 30. Сколь­ко элементов может быть в их:

а) пересечении; б) объединении?

Ответ: а) от 0 до 30; б) от 40 до 70.

“Ёлки” и “Неудержимый”: Некоторые ребята из нашего класса любят ходить в кино. Известно, что 12 ребят смотрели фильм «Ёлки», 9 человек – фильм «Неудержимый», из них 6 смотрели и «Ёлки», и «Неудержимый». Сколько человек смотрели только фильм «Неудержимый»?
Сначала заполняем пересечение. Это будет число 6. Потом заполняем множество ребят, смотревших фильм «Ёлки». Это будет число 6. Так как 6 из двенадцати к тому же ещё смотрели фильм «Неудержимый». После заполняем множество ребят, смотревших фильм «Неудержимый». Это будет число 3. Так как 6 из 9 к тому же ещё смотрели фильм «Ёлки».
Ответ: 3 человека смотрели только фильм «Неудержимый».


20 человек знают английский и 10 – немецкий, из них 5 знают и английский, и немецкий. Сколько человек всего?

Способ 1. С помощью модели «Круги Эйлера» (Приложение 1).

10+20 – 5=25 человек.

Способ 2.

1) 20 – 5 = 15(чел.) – знают только английский язык;

2) 10 – 5 = 5 (чел.) – знают только немецкий язык;

3) 15+5+5 = 25 (чел.) – всего.


15

5

10

А

Можно решать и короче:

  1. 20 – 5 = 15(чел.) – знают только английский язык;

  2. 10+15 = 25 (чел.) – знают немецкий и только английский

2.2. Задачи на пересечение и объединение трёх множеств

В классе всего 36 человек. Учащиеся посещают математический, физический и химический кружки, причем, математический кружок посещают 18 человек, физический – 14 человек, химический – 10 человек. Кроме того, известно, что все три кружка посещают 2 человека, математический и физический -8,математический и химический – 5, физический и химический – 3.

Сколько учеников класса не посещают никаких кружков?

Способ 1. На рисунке большой круг изображает множество всех учеников класса. Внутри этого круга расположены три пересекающихся круга меньшего диаметра: эти круги изображают соответственно множества членов математического, физического и химического кружков. Эти круги обозначены буквами М, Ф, Х.

Общей части всех трех кругов соответствует множество ребят, посещающих все три кружка, поэтому она обозначена МФХ.

Через обозначено множество ребят, посещающих математический и физический кружки, но не посещающих химический кружок. Аналогичным образом обозначены и все остальные области. Здесь для удобства обозначений мы будем отсутствие отмечать чертой над символом.

Теперь обратимся к числовым данным (см. Приложение 2).

В область МФХ впишем число 2, т.к. все три кружка посещают 2 ученика. Далее известно, что ребят, посещающих математический и физический кружки, было 8. Значит, в область МФ надо вписать число 8. Но область МФ состоит из двух частей: и МФХ, причем в МФХ входят 2 человека. Значит, на долю остается 6 человек.

Теперь рассмотрим множество МХ, на которое приходится 5 человек. Эта область также состоит из двух частей. На МФХ приходится 2 человека, значит, на приходится 3.

Рассмотрим теперь множество М, в которое входят 18 учеников. Оно состоит из 4 частей. Количественный состав трех подмножеств мы уже нашли: это 2, 6 и 3. Значит, в четвертое подмножество входит 18 – (2+3+6) = 7 человек.

Рассмотрим множество ФХ, на которое приходится 3 человека. Эта область также состоит из двух частей. На МФХ приходится 2 человека, значит, на приходится 1.

Рассмотрим множество Ф, в которое входят 14 учеников. Оно состоит из 4 частей. Количественный состав трех подмножеств мы уже нашли: это 2, 6 и 1. Значит, в четвертое подмножество входит 14 – (2+1+6) = 5 человек.

36 – (10+7+6+5) = 8 человек. Таким образом, в классе 8 ребят, не посещающих никаких кружков.


М

6

5

7

2

3

1


4

? 8

Способ 2. С помощью модели «Круги Эйлера» (Приложение 1).

Представим множества учащихся, посещаю­щих математический, физический и химический круж­ки, в виде кругов, вырезанных из плотной бумаги. Бу­дем считать, что площадь каждого из этих кругов равна числу учащихся, посещающих соответствующий кру­жок. Наложим круги друг на друга так, чтобы было по­нятно, что есть учащиеся, посещающие один, два или три кружка. Вычислим площадь получившейся фигуры:

14 + 18 + 10 – ((8 + 5 + 3)  2) – 2 = 8 (чел.)— не посещают кружки.

Из 100 туристов, отправляющихся в заграничное путешествие, немецким языком владеют 30 человек, английским – 28, французским – 42. Английским и немецким одновременно владеют 8 человек, английским и французским – 10, немецким и французским – 5, всеми тремя языками – 3. Сколько туристов не владеют ни одним языком?

Всеми тремя языками владеют три туриста, значит, в общей части кругов вписываем число 3. Английским и французским языком владеют 10 человек, а 3 из них владеют еще и немецким. Следовательно, только английским и французским владеют 10-3=7 человек.

Аналогично получаем, что только английским и немецким владеют 8-3=5 человек, а немецким и французским 5-3=2 туриста.

Немецкий знают 30 человек, но 5+3+2=10 из них владеют и другими языками, следовательно, только немецкий знают 20 человек. Аналогично получаем, что одним английским владеют 13 человек,а одним французским – 30.

Всего 100 туристов. 20+13+30+5+7+2+3=80 туристов знают хотя бы один язык, следовательно, 20 человек не владеют ни одним из данных языков.

Заключение

Существует множество приемов, которые используются при решении текстовых логических задач (Приложение 3). Очень часто решение задачи помогает найти рисунок, он делает решение простым и наглядным. Задачи, решаемые с помощью кругов Эйлера, предлагаются на математических олимпиадах, но в школьной программе не отводятся часы на изучение данной темы. Ценность использования кругов Эйлера состоит в том, что решения задач с громоздкими условиями и со многими данными становятся проще.

Подобные задачи часто имеют практический характер, что немаловажно в современной жизни. Они заставляют задумываться, подходить к решению какой-либо проблемы с разных сторон, уметь выбирать из множества способов решения наиболее простой, легкий путь.

Нами созданы модели «Круги Эйлера» для решения логических задач на пересечение двух и трёх множеств, которыми можно пользоваться как на месте (за партой), так и у доски (Приложение 4).

Поиск готовых способов решения выделенных логических задач, самостоятельное описание способа действий при использовании кругов Эйлера для их решения, а также попытки рассмотрения другой формы представления данных условия позволили нам решить поставленные задачи.

Цель была достигнута. С результатами работы были ознакомлены наши одноклассники, что позволило решать логические задачи этого вида не только нам.

Теперь наши одноклассники решают такие задачи, используя не только модели, но и памятку со способом действий, написанных нами.

Теперь мы точно будем знать, сколько друзей нам надо встречать в гости. От 20 до 35! А значит, и за стол всех всё же можно будет посадить.

Данная тема, безусловно, расширяет математический кругозор учащихся, обогащает арсенал средств, используемых в решении разнообразных задач.

Литература

  1. Задачи для внеклассной работы по математике в V – VI классах: Пособие для учителей Текст/ Сост. В.Ю. Сафонова. Под ред. Д.Б. Фукса, А. Л. Гавронского. М.: МИРОС, 1993. с. 42. – ISBN 5-7084-0023-4

  2. Занимательная математика. 5 – 11 классы. Текст: (Как сделать уроки нескучными) / Авт. – сост. Т.Д. Гаврилова. Волгоград: Учитель, 2005. с.32-38. – 10000 экз. –5-7057-0482-8

  3. Депман,И.Я., Виленкин, Н.Я. За страницами учебника математики Пособие для учащихся 5 – 6 кл. Текст/ И.Я Депман. М.: Просвещение, 1999. с. 189 – 191, 231. – 10000 экз. – ISBN 5-09-007107-1

  4. Смыкалова, Е.В. Дополнительные главы по математике для учащихся 5 класса. Текст: СПб: СМИО Пресс, 2009. с.14-20. – 2000 экз. – ISBN 5-7704-0055-2

  5. Фарков, А.В. Математические олимпиады в школе.5–11 классы.Текст / А.В. Фарков. М.: Айрис–пресс, 2007. с. 27, 34, 61. – 7000 экз. – ISBN 978-5-8112-2394-7

  6. Энциклопедия для детей. Т. 11. Математика Текст/ Глав.ред. М.Д. Аксёнова. М.: Аванта +,2001. с. 537 – 542. – 20000экз. – ISBN 5-8483-0015-1

  7. Иванищев, Д. М. Поляна загадок – математика царица.

/

  1. Дистанционная обучающая олимпиада по математике (ДООМ)

/

  1. Сопова, С. С. Диаграмма Эйлера-Вена и “дерево”. Взаимодополнение.

/

Приложение 1

Модель «Круги Эйлера» на пересечение двух множеств

  1. На листе бумаги нарисовать два круга.

  2. Разрезать по пунктирным линиям и получить детали.

  3. На бумаге цвета 1 обвести и вырезать детали № 1 () (), № 2 ().

На бумаге цвета 2 обвести и вырезать детали № 2, № 3 () ().

 – окошко для названия множества,  – окошко для числа

Модель «Круги Эйлера» на пересечение трёх множеств

  1. На листе бумаги нарисовать три круга.

  2. Разрезать по пунктирным линиям и получить детали.

  3. На бумаге цвета 1 обвести и вырезать детали № 5 () (), № 2, № 1, № 4.

На бумаге цвета 2 обвести и вырезать детали № 6 (), (), № 2, № 1, № 3.

На бумаге цвета 3 обвести и вырезать детали № 7 (), (), № 4 (), № 1 (),

№ 3 ().

Приложение 2.

Способ действий при решении задач

на пересечение и объединение трёх множеств с помощью кругов Эйлера

  1. Начертить три пересекающихся круга. Обозначить множества: A, B, C.

  2. Начертить большой круг, в котором окажутся три маленьких. Это общее количество объектов – множество Е.

  3. Начертить отдельное множество D – подмножество множества E Это те, кто не является элементом множеств А, В и С.

  4. Найти часть круга, являющуюся общей для всех трёх множеств (№1) и записать данные.

  5. Найти часть круга, являющуюся общей для двух множеств (№1 и №2) и записать данные в №2.

Найти часть круга, являющуюся общей для двух множеств (№1 и №3) и записать данные в №3.

Найти часть круга, являющуюся общей для двух множеств (№1 и №4) и записать данные в №4.

  1. Найти часть круга, отвечающую за каждое множество в отдельности:

5 = А – (1 + 2 + 4), 6 = В – (1 + 2 + 3), 7 = С – (1 + 3 + 4).

  1. Должно выполняться: 1 + 2 + 3 + 4 + 5 + 6 + 7 + D = E/

  2. Записываем ответ на вопрос задачи.


Приложение 3.

Задача (/). а) На 3 курсе факультета обучается 81 студент. Многие из них выбрали одинаковые дисциплины, посещают одни и те же лекции и хорошо знают друг друга. б) 43 студента посещают лекции по философии, в)32 – по логике и г)41 – по естествознанию. д) Философию и логику выбрали 11 человек. е) Философию и естествознание посещает 21 студент, ж)а логику и естествознание – 16. з) 4 человека выбрали только философию и логику.

Сколько студентов посещают лекции:

1) по всем трём предметам,

2)только по философии и естествознанию,

3)только по логике и естествознанию,

4)только по философии,

5)только по естествознанию,

6)только по логике,

7)не выбрали ни одну из этих дисциплин.

Каждое высказывание из условия записать в виде логического выражения, строго подписывая друг под другом элементы. Решать систему будем с тех уравнений, где меньше всего неизвестных, попарно вычитая уравнения. При решении стремимся убрать как можно больше неизвестных.

1) Возможные варианты перебираем с учетом

а) + + + + + + + = 81

б) + 0 + 0 + + + 0 + + 0 = 43

в) 0 + + 0 + + 0 + + + 0 = 32

г) 0 + 0 + + 0 + + + + 0 = 41

д) 0 + 0 + 0 + + 0 + 0 + + 0 = 11

е) 0 + 0 + 0 + 0 + + 0 + + 0 = 21

ж) 0 + 0 + 0 + 0 + 0 + + + 0 = 16

з) 0 + 0 + 0 + + 0 + 0 + 0 + 0 = 4

2) Четко видно, что = 4. Подписываем под чертой вычисленные значения и убираем использованные уравнения. Ниже приведен подробный ход решения.

а) + + + + + + + = 81

б) + 0 + 0 + + + 0 + + 0 = 43

в) 0 + + 0 + + 0 + + + 0 = 32

г) 0 + 0 + + 0 + + + + 0 = 41

д) 0 + 0 + 0 + + 0 + 0 + + 0 = 11

е) 0 + 0 + 0 + 0 + + 0 + + 0 = 21

ж) 0 + 0 + 0 + 0 + 0 + + + 0 = 16

и) 4

а) + + + + + + + = 81

б) + 0 + 0 + + + 0 + + 0 = 43

в) 0 + + 0 + + 0 + + + 0 = 32

г) 0 + 0 + + 0 + + + + 0 = 41

е) 0 + 0 + 0 + 0 + + 0 + + 0 = 21

ж) 0 + 0 + 0 + 0 + 0 + + + 0 = 16

и) 4 7

а) + + + + + + + = 81

б) + 0 + 0 + + + 0 + + 0 = 43

в) 0 + + 0 + + 0 + + + 0 = 32

г) 0 + 0 + + 0 + + + + 0 = 41

и) 4 14 9 7

а) + + + + + + + = 81

и) 18 12 11 4 14 9 7

0) + + ++ + + + = 81

и) 18 12 11 4 14 9 7 6

Ответ:1) по всем трём предметам, , 7

2)только по философии и естествознанию, , 14

3)только по логике и естествознанию, , 9

4)только по философии, , 18

5)только по естествознанию, , 11

6)только по логике, , 12

7)не выбрали ни одну из этих дисциплин, , 6

Приложение 4

Отчёт о проделанной работе перед коллегами

КОМБИНАТОРИКА. РЕШЕНИЕ ЗАДАЧ С ПОМОЩЬЮ КРУГОВ ЭЙЛЕРА.

КОМБИНАТОРИКА. РЕШЕНИЕ ЗАДАЧ С ПОМОЩЬЮ КРУГОВ ЭЙЛЕРА.

Щербакова А.Ю. 1

1

Коренец Е.И. 1

1

Текст работы размещён без изображений и формул.
Полная версия работы доступна во вкладке “Файлы работы” в формате PDF

         

Именно математика дает

надежнейшие правила:

кто им следует – тому не опасен

обман чувств.

Л. Эйлер

Введение

Во все времена представителям самых различных специальностей приходится решать задачи, в которых рассматриваются те или иные комбинации, составленные из букв, цифр и иных объектов.

Комбинаторика – раздел математики, в которой изучаются вопросы о том, сколько различных комбинаций, подчинённых тем или иным условиям, можно составить из данных объектов. Особая примета комбинаторных задач – это вопрос, который можно сформулировать таким образом, что он начинался бы словами:

Термин «комбинаторика» происходит от латинского слова «combina», что в переводе на русский означает – «сочетать», «соединять».

Выбор объектов и расположением их в том или ином порядке приходится заниматься чуть ли не во всех областях человеческой деятельности, например конструктору, разрабатывающему новую модель механизма, учёному-агроному, планирующему распределение сельскохозяйственных культур на нескольких полях, химику, изучающему строение органических молекул, имеющих данный атомный состав.

Гипотеза работы: Решение комбинаторных задач с помощью кругов Эйлера развивает творческие способности, помогает при решении олимпиадных задач, имеет практическое применение.

Основополагающий вопрос: А все ли я знаю о комбинаторике?

Проблемный вопрос: Может ли помочь комбинаторика в реальной жизни?

Цель работы: изучить решение логических задач путем построения кругов (диаграмм)Эйлера.

Задачи:

  • Познакомиться с историей возникновения науки комбинаторики;

  • Находить возможные комбинации для решения комбинаторных задач

  • Уметь составлять и решать задачи с помощью кругов Эйлера;

  • Поработать с ресурсами Internet;

  • Применять полученные знания в дальнейшем обучении;

  • Расширить и углубить представление о практическом значении математики в жизни;

  • Уметь работать с научно-познавательной литературой, анализировать, делать выводы.

Объект исследования : логические задачи.

Методы:отбор источников информации, изучение материала и анализ его.

Актуальность выбранной темы заключается в необходимости решения комбинаторных задач на уроках математики, применении их в жизни, т.к. они имеют социальную значимость, помогают разобраться в новых веяниях жизни. Основа хорошего понимания комбинаторики – умение считать, думать, рассуждать, находить удачные решения задач.

История комбинаторики

Комбинаторика занимается различного вида соединениями, которые можно образовать из элементов конечного множества. Комбинаторные мотивы можно заметить в символике китайской «Книги Перемен»(Vвек до н.э.). По мнению её авторов, все в мире комбинируется из различных сочетаний мужского и женского начал, а также восьми стихий: земля, горы, вода, ветер, гроза, огонь, облака и небо. Большой интерес математиков вызывали магические квадраты.(см.Приложение 1) Некоторые элементы комбинаторики были известны в Индии еще во II в. до н. э. Индийцы умели вычислять числа, которые сейчас называют «сочетания».Античные греки рассматривали комбинаторные задачи. Хрисипп (IIIв. до н. э.) и Гиппарх(II в.до н.э.) подсчитывали сколько следствий можно получить из 10 аксиом. У Хрисиппа получилось более миллиона. Во все века математики исследовали задачи, связанные с перестановками и сочетаниями, включая перестановки с повторениями. Позднее Д.Кардано провел исследование азартной игры в кости . (Азартными называют те игры, в которых выигрыш зависит не только от умения игрока, но и от случайности). Было замечено, что при многократном бросании однородного кубика (все шесть граней которого отмечены соответственно числами 1, 2, 3, 4, 5, 6) число очков от 1 до 6 выпадают в среднем одинаково часто, иными словами, выражаясь языком математики, выпадение определённого числа очков имеет вероятность, равную 1/6. Аналогично вероятность появления на верхней грани кости чётного числа очков равна 3/6, так как из шести равновозможных случаев чётное число появляется только в трёх. Математически заинтересовались азартной игрой П.Ферма и Б.Паскаль. Помимо азартных игр, комбинаторные методы использовались в криптографии – как для разработки шифров, так и их взломов. Комбинаторика и треугольник Паскаля. Паскаль много занимался биномиальными коэффициентами и открыл их способ вычисления. Число сочетаний можно вычислять не через факториал, а с помощью арифметического треугольника. Строится треугольник: его бедра и вершина состоят из единиц, а в основании каждый элемент строки получается суммированием двух стоящих непосредственно над ним элементов.(см.Приложение 2) Паскаль также как и Лейбниц считается основоположником современной комбинаторики .(см.Приложение 3) А вот сам термин комбинаторика придумал Лейбниц. В 1666г. он опубликовал книгу «Рассуждения о комбинаторном искусстве ». Его ученик – Якоб Бернулли (см.Приложение 4) – основатель теории вероятности, изложил много интересного о комбинаторике. Дал научное обоснование теории сочетаний и перестановок. Изучением размещений занимался Я. Бернулли во второй части своей книги «Искусство предугадывания» в 1713 г., в которой указал формулы для числа размещений из n элементов по k, выводились выражения для степенных сумм .

Позднее обнаружили тесную связь между комбинаторными и аналитическими задачами Абрахам де Муавр Джеймс Стирлинг. Они нашли формулы для нахождения факториала. Окончательно комбинаторику, как раздел математики, оформил в своих трудах Эйлер. Кроме перестановок и сочетаний Эйлер изучал разбиение, а также сочетания и размещения с условиями.

Современным отцом комбинаторики считается Пал Эрдёш. (см.Приложение 5) Он ввел вероятностный анализ. Внимание к комбинаторике повысился во второй половине XX века, с появлением компьютеров.

Многие специалисты в области математики и физики считают, что именно комбинаторная задача может стать толчком в развитии всех технических наук. Некоторые из них всерьез утверждают, что комбинаторика является подспорьем для всех современных наук, особенно космонавтики.

Области применения комбинаторики:

  • учебные заведения ( составление расписаний)

  • сфера общественного питания (составление меню)

  • лингвистика (рассмотрение вариантов комбинаций букв)

  • география (раскраска карт)

  • спортивные соревнования (расчёт количества игр между участниками)

  • производство (распределение нескольких видов работ между рабочими)

  • агротехника (размещение посевов на нескольких полях)

  • азартные игры (подсчёт частоты выигрышей)

  • химия (анализ возможных связей между химическими элементами)

  • экономика (анализ вариантов купли-продажи акций)

  • криптография (разработка методов шифрования)

  • доставка почты (рассмотрение вариантов пересылки)

  • биология (расшифровка кода ДНК)

  • военное дело (расположение подразделений)

  • астрология (анализ расположения планет и созвездий

Теория конфигураций и теория перечисления

Наиболее разработанным разделом комбинаторики является теория конфигураций. Она рассматривает задачи выбора и расположения элементов некоторого множества, в соответствии с заданными правилами. Элементарными комбинаторными конфигурациями являются сочетания, размещения, перестановки. Для подсчёта числа этих конфигураций используются правила суммы и произведения.

Правило суммы:

Если элемент A можно выбрать m способами, а элемент B можно выбрать k способами, то выбор элемента A или B можно осуществить m + k способами.

Правило суммы можно перефразировать на теоретико-множественном языке. Обозначим через | A | число элементов множества A, через A B – объединение множеств A и B, через AxB – декартово произведение множеств A и B. Тогда для непересекающихся множеств A и B выполняется равенство:

| A B | = | A | + | B |.

Обобщением правила суммы является правило произведения.

Правило произведения:

Если элемент A можно выбрать m способами, а после каждого выбора элемента A элемент B можно выбрать k способами, тогда, упорядоченную пару элементов (A, B) можно выбрать m*k способами.

Правило произведения можно распространить на выбор последовательности (x1, x2, …, xn) произвольной конечной длиныn. На теоретико-множественном языке правило произведения формулируется так:

| Aх B | = | A | | B |.

Правило размещения.

Назовём множество, содержащееn элементов, n-множеством.

Последовательность (x1, x2, …, xk ) длины k без повторяющихся элементов из элементов данного n-множества назовём k-размещением.

Обозначим символом число размещений из n по k элементов (от фран. “arrangement” – размещение). Используя правило произведения, вычислим число . Пусть произвольное размещение длины k имеет вид: (x1, x2, …, xk ).

Элемент x1 можно выбрать n способами. После каждого выбора x1 элемент x2 можно выбрать (n – 1) способами. После каждого выбора элементов x1 и x2 элемент x3 можно выбрать (n – 2) способами, и т.д. После каждого выбора элементов x1, x2, …, xk-1 элемент xk можно выбрать (n -(k – 1)) = (n – k + 1) способами. Тогда, по правилу произведения, последовательность (x1; x2; , …, xk ) можно выбрать числом способов, равным

n(n – 1)(n – 2) … (n – k + 1) = (1.1)

Произведение в левой части равенства (1.1) умножим и разделим на (n – k)!, получим:

. (1.2)

Если в форуме (1.2) k = n, то есть число Pn перестановок из n элементов

Pn = n! (от “permutation”- перестановка).

Правило сочетания.

k-подмножество данного n-множества называется k-сочетанием.

Обозначим через число k-сочетаний из данныхn элементов. Формулу для числа получим, рассуждая следующим образом. Если каждое сочетание упорядочить всеми возможными способами, то получим все k-последовательностей изn элементов, без повторений, то есть все k-размещения. Иными словами, Откуда: (1.3) или Предполагая, что n и k – целые положительные числа и 0!=1, сформулируем основные свойства сочетаний.

Основные свойства сочетаний.
  1. Условились, что

Как выбрать формулу? (см.Приложение 6) Сводка формул для всех видов соединений. (см.Приложение 7)

Сочетания и размещения широко используются при вычислении классической вероятности случайных событий.

Пример. В корзине находятся 20 орехов, из которых 7 грецких. Наудачу выбирают 5 орехов. Найти вероятность того, что среди выбранных орехов содержатся 2 грецких.

Решение. Число исходов опыта . Случайное событие A – среди пяти выбранных орехов содержатся 2 грецких ореха. Число исходов, благоприятствующих событию A, равно: . Искомая вероятность .

Задачи.

  1. Найти вероятность того, что случайно выбранное 5-значное (десятичное) число не содержит цифры 5.

  2. Предприятие располагает 5 вакансиями для мужчин, 5 вакансиями для женщин и 4 вакансиями для работников любого пола. В отдел кадров предприятия обратилось 20 человек, среди которых 12 мужчин и 8 женщин. Сколькими способами предприятие может заполнить имеющиеся вакансии?

  3. В классе 25 учеников, из которых 13 юношей и 12 девушек. Сколькими способами 25 учеников могут встать в шеренгу так, чтобы юноши после удаления из строя девушек, оказались построенными по росту; аналогично девушки после удаления из строя юношей оказались построенными по росту?

Круги Эйлера

Круги Эйлера – это геометрическая схема, которая помогает находить и/или делать более наглядными логические связи между явлениями и понятиями. А также помогает изобразить отношения между каким-либо множеством и его частью.

На рисунке представлено множество – все возможные игрушки. Некоторые из игрушек являются конструкторами – они выделены в отдельный овал. Это часть большого множества «игрушки» и одновременно отдельное множество (ведь конструктором может быть и «Лего», и примитивные конструкторы из кубиков для малышей). Какая-то часть большого множества «игрушки» может быть заводными игрушками. Они не конструкторы, поэтому мы рисуем для них отдельный овал. Желтый овал «заводной автомобиль» относится одновременно к множеству «игрушки» и является частью меньшего множества «заводная игрушка». Поэтому и изображается внутри обоих овалов сразу.

Автор метода – ученый Леонард Эйлер (1707-1783) (см.Приложение 8) . Он так и говорил о названных его именем схемах: «круги подходят для того, чтобы облегчить наши размышления». Эйлер считается немецким, швейцарским и даже российским математиком, механиком и физиком. Дело в том, что он много лет проработал в Петербургской академии наук и внес существенный вклад в развитие российской науки. Метод Эйлера получил заслуженное признание и популярность. И после него немало ученых использовали его в своей работе, а также видоизменяли на свой лад. Например, чешский математик Бернард Больцано использовал тот же метод, но с прямоугольными схемами. Круги Эйлера имеют прикладное назначение. С их помощью на практике решаются задачи на объединение или пересечение множеств в математике, логике. Круги Эйлера можно разделить их на те, что описывают объединение каких-то понятий и описывают пересечение множеств по какому-то признаку. (см.Приложение9)

Пример пересечения – какую профессию выбрать? Нарисую схему в виде кругов Эйлера. Схема сразу расставит все по местам и поможет определиться с выбором. То, что окажется на пересечении всех трех кругов, и есть профессия, которая не только сможет прокормить, но и будет нравиться.

Чертеж, вроде этого, поможет определиться с выбором:

Рассмотрим несколько примеров задач, которые можно решить с помощью кругов Эйлера.

В языке запросов поискового сервера для обозначения логической операции «ИЛИ» используется символ «|», а для логической операции «И» – символ «&».

В таблице приведены запросы и количество найденных по ним страниц некоторого сегмента сети интернет.

Запрос

Найдено страниц (в тысячах)

Крейсер | Линкор

7000

Крейсер

4800

Линкор

4500

Какое количество страниц (в тысячах) будет найдено по запросу Крейсер & Линкор?

Считается, что все вопросы выполняются практически одновременно, так что набор страниц, содержащих все искомые слова, не изменялся за время выполнения запросов.

Решение:

При помощи кругов Эйлера изобразим условия задачи. При этом цифры 1, 2 и 3 используем, чтобы обозначить полученные в итоге области.

Опираясь на условия задачи, составим уравнения:

  1. Крейсер | Линкор: 1 + 2 + 3 = 7000

  2. Крейсер: 1 + 2 = 4800

  3. Линкор: 2 + 3 = 4500

Чтобы найти Крейсер & Линкор (обозначенный на чертеже как область 2), подставим уравнение (2) в уравнение (1) и выясним, что:

4800 + 3 = 7000, откуда получаем 3 = 2200.

Теперь этот результат мы можем подставить в уравнение (3) и выяснить, что:

2 + 2200 = 4500, откуда 2 = 2300.

Ответ: 2300 – количество страниц, найденных по запросу Крейсер & Линкор.

Вывод

Круги Эйлера помогают быстро и просто решить даже достаточно сложные или просто запутанные на первый взгляд задачи. Для этого необходимо запомнить порядок этапов :

  1. Записать краткое условие.

  2. Выполнить рисунок.

  3. Записать данные в круги Эйлера.

  4. Анализировать, рассуждать и записывать результаты в части круга.

  5. Записать ответ.

Задачи:

1. Сколько существует натуральных чисел, меньших 1000, которые делятся на 3 , но не делятся на 2 и на 5?

2. Сколько существует различных десятизначных чисел, состоящих только из нулей и единиц, которые содержат не более трех единиц ?

3. Биатлонист проходит четыре огневые точки, на каждой он делает по 5 выстрелов. Сколько существует различных способов промахнуться не более 4?

4. По результатам одного социологического исследования, было установлено, что из 200 людей смотрящих телевизор, 110 человек смотрят спортивную передачу, 120 – комедии, 85 предпочитаю драмы, 50 смотрят драмы и спорт, 70 – комедии и спорт, 55 смотрят комедии и драмы и 30 человек смотрят все три вида передач. Сколько человек, смотрят спорт или комедии или драмы? сколько человек не смотрят ничего из вышеперечисленного?

5. Человек имеет 10 друзей и течение нескольких дней, приглашает некоторых из них в гости, так что компания, ни разу не повторяется. Сколько дней он может так делать?

6. Найти количество трехзначных чисел, которые делятся на 7, но не делятся на 2 и на 5.

7. На данный момент, в классе 20 учеников, получивших сначала учебного года, хотя одну двойку, 17 учеников, получивших не менее двух двоек, 8 учеников, получивших не менее трех двоек, три ученика получивших не менее 4 двоек, один ученик получивший 5 двоек, больше пяти двое нет ни у кого. Сколько всего двое в журнале?

Задача 1.

Решение:

Определение: множество А называется подмножество множества В, если каждый элемент множества А принадлежит множеству В.

Если в некоторой задаче считается что элементы принадлежат некоторым множествам, то это множество называется универсальным.

Например, в задаче № 1 универсальным множеством можно считать множество чисел, от 1 до 999.

A = количество элементов во множестве А

В = 333

Если число делится на 2 и 3, то число делится на 6

A В = 166

Если число делится на 3 и 5, то число делится на 15

В D = 66

При таком подсчете, мы дважды посчитали числа, которые входят во множество

АВD = 33

(В/А)/D = В – AВ – ВD + AВD = 333 – 166 – 66 + 33 = 134

Задача 2

Определение: Правило суммы.

Все множества способов подсчета можно разбить на пересекающиеся множества. Тогда общее количество способов вычисляется как сумма множеств.

С = = 1-0 единиц

С = = 10-1 единица

С = = 45

С = = 120

120+45+10+1=176

Задача 3.

С = = 1 способ – 0 промахов

C = = 20 способов – 1 промах

С = = 190 способов – 2 промаха

С = =1140 способов – 3 промаха

С = =4845 способов – 4 промаха

Всего 1 + 20 + 190 +1140 + 4845= 6196 способов

Задача 4.

Формула включений и исключений

А ВD = A + В +D – AВ – AD – ВD – AВD

а) А ВD = 110 + 120 + 85 – 70 -55 – 50 + 30 =170

б)200-170=30 человек ничего не смотрят

Задача 5.

Составим таблицу друзей

С = = 10 компаний из 1 человека

С = = 45 компаний из 2 человек

С = = 120 компаний из 3 человек

С = = 210 компаний из 4 человек

С = = 252 компании из 5 человек

С ==210 компаний из 6 человек

С = = 120 компаний из 7 человек

С = =45 компаний из 8 человек

С = =10 компаний из 9 человек

С = = 1 компания из 10 человек

Итого 10+45+120+210+252+210+120+45+10+1= 1023 способов

Задача 6.

Всего 900 трехзначных чисел.

A = 900:7=128 чисел

АВ =900:14=64 числа

АD = 900: 35=25 чисел

АВD =900:70=12 чисел

А = А – АВ – АD + АВD = 128 – 64 – 25 + 12 = 51 число

Задача 7.

Найдем количество учеников, получивших ровно четыре двойки.

  1. 3 – 1= 2 ученика получили 4 двойки

Теперь узнаем количество учеников , получивших ровно три двойки, ровно две двойки и ровно одну двойку.

  1. 8 – 2 – 1 = 5 учеников получили 3 двойки

  2. 17 – 5 – 2 – 1 = 9 учеников получили 2 двойки

  3. 20 – 9 – 5 – 2 – 1 = 3 ученика получили 1 двойку

  4. 1×3 + 2×9 + 3×5 + 4×2 + 5×1 = 49 двоек в журнале

Вывод

Для решения данных логических задач, использовала круги Эйлера, что позволило успешно решить поставленные задачи. Этот способ показался мне удобным и надежным, так как он упрощает путь к решению задачи, делая его наглядным.

Заключение

В процессе изучения данной темы, я научилась грамотно оперировать такими понятиями как «множество», «объединение множеств», «пересечение множеств», «разность множеств» и использовать их при решении задач. В процессе решения задач я расширила свои знания по математике, познакомилась с ещё одним способом решения задач, который был мне мало знаком. Для решения задач с помощью кругов Эйлера можно воспользоваться алгоритмом, состоящим из нескольких этапов.

Применение кругов Эйлера позволяет легко решить задачи, которые обычным путем разрешимы составлением сложных уравнений. Моя гипотеза подтвердилась. Решения задач с громоздкими условиями и со многими данными просты и не требуют особых умозаключений. Применение кругов Эйлера придает задачам наглядность и простоту.

Практическая значимость заключается в расширении возможностей при решении логических задач. Пригодится для решения задач занимательного характера, позволит применять методы и правила для решения нетрадиционных задач. Приобретенные сведения и знания способствуют повышению интеллектуального развития, помогают развить умение наблюдать и анализировать.

Круги Эйлера – не просто занимательная и интересная штука, но и весьма полезный метод решения задач. Причем не только абстрактных задач на школьных уроках, но и вполне себе житейских проблем. Они заставляют задумываться, подходить к решению какой-либо проблемы с разных сторон, уметь выбирать из множества способов решения наиболее простой, легкий путь. Сам Леонард Эйлер говорил: «круги подходят для того, чтобы облегчить наши размышления».

Список использованной литературы

  1. Галеева Р. А. Тренируем мышление. Задачи на сообразительность / Р. А. Галеева, Г. С. Курбанов, И. В. Мельченко – Изд. 2 – е – Ростов н/Д: Феникс, 2006.

  2. Игнатьев. Е.И. В царстве смекалки, или Арифметика для всех: Книга для семьи и школы. Опыт математической хрестоматии в 3 книгах/Худож. Н.Я. Бойко. – Ростов н/Д: Кн. Изд-во, 1995.

  3. Рыбников К.А.Комбинаторный анализ. Очерки истории.-М.: Изд.мехмата МГУ1996.-124с.

  4. История математики под редакцией Юшкевича А.П. М.: Наука Том 1.С древнейших времен до начала Нового времени.1970.

  5. Нагибин Ф.Ф., Канин Е.С. Математическая шкатулка: Пособие для учащихся 4-8 кл. сред. Шк. – 5-е изд. – М.: Просвещение, 1988.

  6. Увлекательные логические задачки, которые будут интересны детям и взрослым. http://logika.vobrazovanie.ru

Приложение

Приложение 1.

Приложение 2.

Приложение 3.

Приложение 4.

Якоб Бернулли.

Приложение 5.

Математика – это орудие, с помощью которого человек познает и покоряет себе окружающий мир.

Пал Эрдеш.

Приложение 6.

Приложение 7.

Приложение 8.

Приложение 9.

Просмотров работы: 5866

Что такое круги эйлера в информатике. Презнтаця по математике на тему “круги эйлера

Каждый предмет или явление обладает некими свойствами (признаками).

Получается, что составить понятие об объекте означает, прежде всего, умение отличить его от других сходных с ним объектов.

Можно сказать, что понятие – это мысленное содержание слова.

Понятие – это форма мысли, отображающая предметы в их наиболее общих и существенных признаках.

Понятие – это форма мысли, а не форма слова, так как слово лишь метка, которой мы помечаем ту или иную мысль.

Слова могут быть различны, но при этом обозначать одно и то же понятие. По-русски – «карандаш», по-английски – «pencil», по-немецки – bleistift. Одна и та же мысль в разных языках имеет разное словесное выражение.

ОТНОШЕНИЯ МЕЖДУ ПОНЯТИЯМИ. КРУГИ ЭЙЛЕРА.

Понятия, имеющие в своих содержаниях общие признаки, называются СРАВНИМЫМИ («адвокат» и «депутат»; «студент» и «спортсмен»).

В противном случае, понятия считаются НЕСРАВНИМЫМИ («крокодил» и «блокнот»; «человек» и «пароход»).

Если кроме общих признаков понятия имеют и общие элементы объёма, то они называются СОВМЕСТИМЫМИ .

Существует шесть видов отношений между сравнимыми понятиями. Отношения между объёмами понятий удобно обозначать с помощью кругов Эйлера (круговые схемы, где каждый круг обозначает объём понятия).

ВИД ОТНОШЕНИЯ МЕЖДУ ПОНЯТИЯМИИЗОБРАЖЕНИЕ С ПОМОЩЬЮ КРУГОВ ЭЙЛЕРА
РАВНОЗНАЧНОСТЬ (ТОЖДЕСТВЕННОСТЬ) Объёмы понятий полностью совпадают. Т.е. это понятия, которые различаются по содержанию, но в них мыслятся одни и те же элементы объёма. 1) А – Аристотель В – основатель логики 2) А – квадрат В – равносторонний прямоугольник
ПОДЧИНЕНИЕ (СУБОРДИНАЦИЯ) Объём одного понятия полностью входит в объём другого, но не исчерпывает его. 1) А – человек В – студент 2) А – животное В – слон
ПЕРЕСЕЧЕНИЕ (ПЕРЕКРЕЩИВАНИЕ) Объёмы двух понятий частично совпадают. То есть понятия содержат общие элементы, но и включают элементы, принадлежащие только одному из них. 1) А – юрист В – депутат 2) А – студент В – спортсмен
СОПОДЧИНЕНИЕ (КООРДИНАЦИЯ) Понятия, не имеющие общих элементов, полностью входят в объём третьего, более широкого понятия. 1) А – животное В – кот; С – собака; D – мышь 2) А – драгоценный металл В – золото; С – серебро; D – платина
ПРОТИВОПОЛОЖНОСТЬ (КОНТРАРНОСТЬ) Понятия А и В не просто включены в объём третьего понятия, а как бы находятся на его противоположных полюсах. То есть, понятие А имеет в своём содержании такой признак, которых в понятии В заменён на противополжный. 1) А – белый кот; В – рыжий кот (коты бывают и чёрными и серыми) 2) А – горячий чай; холодный чай (чай может быть и тёплым) Т.е. понятия А и В не исчерпывают всего объёма понятия, в которое они входят.
ПРОТИВОРЕЧИЕ (КОНТРАДИКТОРНОСТЬ) Отношение между понятиями, одно из которых выражает наличие каких-либо признаков, а другое – их отсутствие, то есть просто отрицает эти признаки, не заменяя их никакими другими. 1) А – высокий дом В – невысокий дом 2) А – выигрышный билет В – невыигрышный билет Т.е. понятия А и не-А исчерпывают весь объём понятия, в которое они входят, так как между ними нельзя поставить никакое дополнительное понятие.

Упражнение : Определите вид отношений по объёму приведённых ниже понятий. Изобразите их с помощью кругов Эйлера .

1) А – горячий чай; В – холодный чай; С – чай с лимоном

Горячий чай (В) и холодный чай (С) – находятся в отношении противоположности.

Чай с лимоном (С) может быть как горячим,

так и холодным, но может быть и, например, тёплым.

2) А – деревянный; В – каменный; С – строение; D – дом.

Всякое ли строение (С) – дом (D)? – Нет.

Всякий ли дом (D) – строение (С)? – Да.

Что-то деревянное (А) обязательно ли дом (D) или строение (С) – Нет.

Но можно найти деревянное строение (например, будка),

также можно найти деревянный дом.

Что-то каменное (В) не обязательно дом (D) или строение (С).

Но может быть и каменное строение, и каменный дом.

3) А – российский город; В – столица России;

С – Москва; D – город на Волге; Е – Углич.

Столица России (В) и Москва (С) – один и тот же город.

Углич (Е) является городом на Волге (D).

При этом, Москва, Углич, как и любой город на Волге,

являются российскими городами (А)

Если Вы считаете, что ничего не знаете о таком понятии, как круги Эйлера, то вы глубоко заблуждаетесь. Еще из младшей школы известны схематические изображения, или кружки, позволяющие наглядно осмыслить взаимоотношения между понятиями и элементами системы.

Метод, придуманный Леонардом Эйлером, использовался ученым для решения сложных математических задач. Кругами он изображал множества и сделал эту схему основой такого понятия, как символическая . Метод призван максимально упростить рассуждения, направленные на решении той или иной задачи, именно поэтому методика активно используется как в младшей школе, так и в академической среде. Интересно, что подобный подход был ранее использован немецким философом Лейбницем, а позже был подхвачен и применен в различных модификациях известными умами в области математики. Например, прямоугольные схемы чешского Больцано, Шредера, Венна, известного созданием популярной диаграммы, основанной на этом простом, но удивительно действенном методе.

Круги являются основой так называемых «наглядных интернет мемов», которые основаны на схожести признаков отдельных множеств. Забавно, наглядно, а главное понятно.

Круги мысли

Круги позволяют наглядно описать условия задачи и мгновенно принять верное решение, или выявить направление движение в сторону правильного ответа. Как правило, круги Эйлера используются для решения логико-математических задач, связанных с множествами, их объединениями или частичными наложениями. В пересечение кругов попадают объекты, обладающие свойствами каждого из изображенных кружком множеств. Объекты, не вошедшие в множество, находятся за пределами того или иного круга. Если понятия абсолютно равнозначны, они обозначаются одним кругом, представляющим собой объединение двух множеств, имеющих равные свойства и объемы.

Логика взаимосвязей

Используя круги Эйлера, вы можете решить ряд бытовых задач и даже определиться с выбором будущей профессии, стоит лишь проанализировать свои возможности и желания и выбрать их максимальное пересечение.

Теперь становится ясно, что круги Эйлера вовсе не абстрактное математическое и философское понятие из разряда теоретических знаний, они имеют весьма прикладное и практическое значение, позволяя разобраться не только с простейшими математическими проблемами, но и решить важные жизненные дилеммы наглядным и понятным каждому способом.

При решении многих задач, связанных с множествами, незаменимым оказывается приём, основанный на использовании так называемых «кругов Эйлера». Эти диаграммы впервые появились в работах одного из величайших математиков в истории Леонарда Эйлера, который в течение продолжительного времени жил и работал в России и был членом Петербургской академии наук. Использование кругов Эйлера добавляет наглядности при решении сложных задач, делая многие вещи буквально очевидными. Предлагаю вам в этом убедиться самостоятельно на примере решения следующей задачи.

Пример решения задачи с помощью кругов Эйлера

Тут нужно понимать, что если сказано, что «42 человека используют метро», то это вовсе не означает, что кроме метро они не используют никаких других видов транспорта. Кто-нибудь из них может быть и использует. Может быть ещё какой-то один вид транспорта, трамвай или автобус. А может и сразу оба! Вопрос задачи как раз и состоит в том, чтобы посчитать людей, которые используют все три вида транспорта.

С первого взгляда даже непонятно, с чего начинать решение. Но если немного поразмыслить, становится ясно, что действовать нужно по следующему алгоритму. Будем стараться расписать всех людей (58 человек) через известные из условия данные. Нам известно, что автобус используют 44 человека. Прибавим к этому количество людей, которые используют метро. Их всего 42 человек. С помощью кругов Эйлера эту операцию можно изобразить наглядно в следующем виде:

То есть пока что мы имеем дело с выражением 58 = 44 + 42… Знак «…» означает, что выражение ещё не закончено. Проблема в том, что мы посчитали людей на пересечении этих кругов дважды. Соответствующая область на диаграмме выделена тёмно-зелёным цветом. Поэтому один раз их нужно вычесть. Это люди, которые пользуются автобусом и метро. Их, как известно, 31. То есть наше «неоконченное» выражение принимает вид: 58 = 44 + 42 — 31… И на диаграмме при этом пропадает тёмно-зелёный цвет:

Пока всё хорошо. Прибавляем теперь людей, которые ездят на трамвае. Таких людей 32. Выражение принимает вид: 58 = 44 + 42 — 31 + 32… Диаграмма с кругами Эйлера, в свою очередь, становится следующей:

К счастью в незакрашенной области как раз и находятся те люди, число которых нам нужно посчитать. Действительно, эти бедняги используют ежедневно все три вида транспорта для того, чтобы добраться до работы, ведь они находятся на пересечении всех трёх множеств. Обозначим количество этих бедолаг за . Тогда диаграмма примет следующий вид:

А уравнение станет следующим:

Расчёты дают . Это и есть ответ к задаче. Столько людей используют все три вида транспорта каждый день, чтобы добраться на работу.

Вот такое вот простое решение. Фактически, в одно уравнение. Просто удивительно, не правда ли?! А теперь представьте, как пришлось бы решать эту задачу без использования кругов Эйлера. Это было бы настоящее мучение. Так что в очередной раз убеждаемся, что любые методы визуализации чрезвычайно полезны при решении задач по математике. Используйте их, это поможет вам в решении сложных задач как на олимпиадах, так и на вступительных экзаменах по математике в лицеи и вузы.

Чтобы проверить, хорошо ли вы поняли решение данной задачи, ответьте на следующие вопросы:

  1. Сколько человек используют только один вид транспорта для того, чтобы добраться до работы?
  2. Сколько человек используют для этого ровно два вида транспорта?

Свои ответы и варианты решения присылайте в комментариях.

Материал подготовил , Сергей Валерьевич

Если вы думаете, что ничего не знаете о кругах Эйлера, вы ошибаетесь. На самом деле вы наверняка не раз с ними сталкивались, просто не знали, как это называется. Где именно? Схемы в виде кругов Эйлера легли в основу многих популярных интернет-мемов (растиражированных в сети изображений на определенную тему).

Давайте вместе разберемся, что же это за круги, почему они так называются и почему ими так удобно пользоваться для решения многих задач.

Происхождение термина

– это геометрическая схема, которая помогает находить и/или делать более наглядными логические связи между явлениями и понятиями. А также помогает изобразить отношения между каким-либо множеством и его частью.

Пока не очень понятно, верно? Посмотрите на этот рисунок:

На рисунке представлено множество – все возможные игрушки. Некоторые из игрушек являются конструкторами – они выделены в отдельный овал. Это часть большого множества «игрушки» и одновременно отдельное множество (ведь конструктором может быть и «Лего», и примитивные конструкторы из кубиков для малышей). Какая-то часть большого множества «игрушки» может быть заводными игрушками. Они не конструкторы, поэтому мы рисуем для них отдельный овал. Желтый овал «заводной автомобиль» относится одновременно к множеству «игрушки» и является частью меньшего множества «заводная игрушка». Поэтому и изображается внутри обоих овалов сразу.

Ну что, так стало понятнее? Именно поэтому круги Эйлера – это тот метод, который наглядно демонстрирует: лучше один раз увидеть, чем сто раз услышать. Его заслуга в том, что наглядность упрощает рассуждения и помогает быстрее и проще получить ответ.

Автор метода – ученый Леонард Эйлер (1707-1783). Он так и говорил о названных его именем схемах: «круги подходят для того, чтобы облегчить наши размышления». Эйлер считается немецким, швейцарским и даже российским математиком, механиком и физиком. Дело в том, что он много лет проработал в Петербургской академии наук и внес существенный вклад в развитие российской науки.

До него подобным принципом при построении своих умозаключений руководствовался немецкий математик и философ Готфрид Лейбниц.

Метод Эйлера получил заслуженное признание и популярность. И после него немало ученых использовали его в своей работе, а также видоизменяли на свой лад. Например, чешский математик Бернард Больцано использовал тот же метод, но с прямоугольными схемами.

Свою лепту внес также немецкий математике Эрнест Шредер. Но главные заслуги принадлежат англичанину Джону Венну. Он был специалистом в логике и издал книгу «Символическая логика», в которой подробно изложил свой вариант метода (использовал преимущественно изображения пересечений множеств).

Благодаря вкладу Венна метод даже называют диаграммами Венна или еще Эйлера-Венна.

Зачем нужны круги Эйлера?

Круги Эйлера имеют прикладное назначение, то есть с их помощью на практике решаются задачи на объединение или пересечение множеств в математике, логике, менеджменте и не только.

Если говорить о видах кругов Эйлера, то можно разделить их на те, что описывают объединение каких-то понятий (например, соотношение рода и вида) – мы их рассмотрели на примере в начале статьи.

А также на те, что описывают пересечение множеств по какому-то признаку. Таким принципом руководствовался Джон Венн в своих схемах. И именно он лежит в основе многих популярных в интернете мемов. Вот вам один из примеров таких кругов Эйлера:

Забавно, правда? И главное, все сразу становится понятно. Можно потратить много слов, объясняя свою точку зрения, а можно просто нарисовать простую схему, которая сразу расставит все по местам.

Кстати, если вы не можете определиться, какую профессию выбрать, попробуйте нарисовать схему в виде кругов Эйлера. Возможно, чертеж вроде этого поможет вам определиться с выбором:

Те варианты, которые окажутся на пересечении всех трех кругов, и есть профессия, которая не только сможет вас прокормить, но и будет вам нравиться.

Решение задач с помощью кругов Эйлера

Давайте рассмотрим несколько примеров задач, которые можно решить с помощью кругов Эйлера.

Вот на этом сайте – http://logika.vobrazovanie.ru/index.php?link=kr_e.html Елена Сергеевна Саженина предлагает интересные и несложные задачи, для решения которых потребуется метод Эйлера. Используя логику и математику, разберем одну из них.

Задача про любимые мультфильмы

Шестиклассники заполняли анкету с вопросами об их любимых мультфильмах. Оказалось, что большинству из них нравятся «Белоснежка и семь гномов», «Губка Боб Квадратные Штаны» и «Волк и теленок». В классе 38 учеников. «Белоснежка и семь гномов» нравится 21 ученику. Причем трем среди них нравятся еще и «Волк и теленок», шестерым – «Губка Боб Квадратные Штаны», а один ребенок одинаково любит все три мультфильма. У «Волка и теленка» 13 фанатов, пятеро из которых назвали в анкете два мультфильма. Надо определить, скольким же шестиклассникам нравится «Губка Боб Квадратные Штаны».

Решение:

Так как по условиям задачи у нас даны три множества, чертим три круга. А так как по ответам ребят выходит, что множества пересекаются друг с другом, чертеж будет выглядеть так:

Мы помним, что по условиям задачи среди фанатов мультфильма «Волк и теленок» пятеро ребят выбрали два мультфильма сразу:

Выходит, что:

21 – 3 – 6 – 1 = 11 – ребят выбрали только «Белоснежку и семь гномов».

13 – 3 – 1 – 2 = 7 – ребят смотрят только «Волк и теленок».

Осталось только разобраться, сколько шестиклассников двум другим вариантам предпочитает мультфильм «Губка Боб Квадратные Штаны». От всего количества учеников отнимаем всех тех, кто любит два других мультфильма или выбрал несколько вариантов:

38 – (11 + 3 + 1 + 6 + 2 + 7) = 8 – человек смотрят только «Губка Боб Квадратные Штаны».

Теперь смело можем сложить все полученные цифры и выяснить, что:

мультфильм «Губка Боб Квадратные Штаны» выбрали 8 + 2 + 1 + 6 = 17 человек. Это и есть ответ на поставленный в задаче вопрос.

А еще давайте рассмотрим задачу , которая в 2011 году была вынесена на демонстрационный тест ЕГЭ по информатике и ИКТ (источник – http://eileracrugi.narod.ru/index/0-6).

Условия задачи:

В языке запросов поискового сервера для обозначения логической операции «ИЛИ» используется символ «|», а для логической операции «И» – символ «&».

В таблице приведены запросы и количество найденных по ним страниц некоторого сегмента сети интернет.

ЗапросНайдено страниц (в тысячах)
Крейсер | Линкор7000
Крейсер4800
Линкор4500

Какое количество страниц (в тысячах) будет найдено по запросу Крейсер & Линкор ?

Считается, что все вопросы выполняются практически одновременно, так что набор страниц, содержащих все искомые слова, не изменялся за время выполнения запросов.

Решение:

При помощи кругов Эйлера изобразим условия задачи. При этом цифры 1, 2 и 3 используем, чтобы обозначить полученные в итоге области.

Опираясь на условия задачи, составим уравнения:

  1. Крейсер | Линкор: 1 + 2 + 3 = 7000
  2. Крейсер: 1 + 2 = 4800
  3. Линкор: 2 + 3 = 4500

Чтобы найти Крейсер & Линкор (обозначенный на чертеже как область 2), подставим уравнение (2) в уравнение (1) и выясним, что:

4800 + 3 = 7000, откуда получаем 3 = 2200.

Теперь этот результат мы можем подставить в уравнение (3) и выяснить, что:

2 + 2200 = 4500, откуда 2 = 2300.

Ответ: 2300 – количество страниц, найденных по запросу Крейсер & Линкор.

Как видите, круги Эйлера помогают быстро и просто решить даже достаточно сложные или просто запутанные на первый взгляд задачи.

Заключение

Полагаю, нам удалось убедить вас, что круги Эйлера – не просто занимательная и интересная штука, но и весьма полезный метод решения задач. Причем не только абстрактных задач на школьный уроках, но и вполне себе житейских проблем. Выбора будущей профессии, например.

Вам еще наверняка будет любопытно узнать, что в современной массовой культуре круги Эйлера нашли отражение не только в виде мемов, но и в популярных сериалах. Таких, как «Теория большого взрыва» и «4исла».

Используйте это полезный и наглядный метод для решения задач. И обязательно расскажите о нем друзьям и одноклассникам. Для этого под статьей есть специальные кнопки.

сайт, при полном или частичном копировании материала ссылка на первоисточник обязательна.

Диаграммы Венна

и диаграммы Эйлера, объясненные на примерах

Диаграммы

Венна и диаграммы Эйлера выглядят очень похоже, поэтому понятно, что многих людей смущает понимание разницы. Хотя оба типа диаграмм основаны на теории множеств, есть некоторые тонкие различия, которые делают их уникальными. Надеюсь, эта статья развеет ваши сомнения относительно диаграмм Венна и диаграмм Эйлера, и я приведу несколько примеров, чтобы сделать ее более ясной.

Венн против Эйлера: определение

Как я упоминал ранее, оба набора диаграмм основаны на теории множеств. Диаграмма Венна показывает все возможные логические отношения между набором наборов. Но диаграмма Эйлера показывает только отношения, существующие в реальном мире.

Диаграммы Венна и

Диаграммы Эйлера Примеры

Начнем с очень простого примера. Давайте рассмотрим надмножество животных с млекопитающими и птицами как подмножества. Диаграмма Венна показывает пересечение двух множеств, хотя такой возможности не существует в реальном мире.Диаграмма Эйлера, с другой стороны, не показывает пересечения.

Диаграммы Венна показывают все возможные комбинации, даже если они не существуют в реальном сценарии.

Теперь давайте рассмотрим более сложный пример с колодой карт. Опять же, важно помнить о различии между двумя типами диаграмм: всех возможных комбинаций и реальных комбинаций. Давайте возьмем карты в качестве расширенного набора, а черные карты, красные карты и ромбы – в качестве подмножества.

Как одни и те же данные представлены по-разному с помощью диаграмм Венна и диаграмм Эйлера

Как показано в приведенном выше примере, диаграммы Венна показывают четыре пересечения, для которых нет данных, потому что они должны отображать все возможные комбинации.

Существуют различные методы преобразования диаграмм Венна в диаграммы Эйлера и наоборот. Ознакомьтесь с этой замечательной вики-статьей о диаграммах Эйлера, в которой объясняются некоторые методы, которые вы можете использовать для преобразования диаграмм Венна в диаграммы Эйлера.Я надеюсь, что приведенные выше примеры помогли вам развеять ваши сомнения относительно диаграмм Венна и диаграмм Эйлера. Если у вас есть какие-либо вопросы, не стесняйтесь задавать их в разделе комментариев.

Рисуете ли вы диаграммы Венна или диаграммы Эйлера, Creately предоставит вам все необходимые инструменты. Вы можете быстро начать работу, используя шаблоны диаграмм Венна, доступные нашим пользователям, или начать с нуля в области рисования. Если вы хотите добавить значки и изображения на диаграмму Венна, это можно легко сделать с помощью встроенного поиска изображений Google, доступного на левой боковой панели.Благодаря такому количеству супер крутых функций вы не ошибетесь с Creately.

Присоединяйтесь к более чем тысячам организаций, которые используют Creately для мозгового штурма, планирования, анализа и успешного выполнения своих проектов.

Начните здесь

Диаграммы Эйлера и Венна: они не только для развлечения

(источник) К тому времени, когда вы дочитаете этот пост в блоге, вы должны быть в третьей группе. Диаграммы Венна и Эйлера (произносится как «масленка») невероятно популярны в Интернете как забавные диаграммы.Они предлагают простой способ изобразить концепции теории множеств. Итак, в чем разница между ними? Почему они смешные? Полезны ли они для реальных данных? Оба типа диаграмм используются для отображения концепций теории множеств:

Union – комбинация двух наборов. В диаграммах Венна и Эйлера.
Intersection – Входит в оба из двух наборов. В диаграммах Венна и Эйлера.
Разница – Все, кроме пересечения двух наборов. В диаграммах Венна и Эйлера.
Относительное дополнение – В одном наборе, а не в другом. В диаграммах Венна и Эйлера.
Абсолютное дополнение – Все, чего нет в другом наборе. Только в диаграммах Эйлера.
Подмножество – Набор, полностью содержащийся в другом наборе. Только в диаграммах Эйлера.
Disjoint – Два набора без общих элементов. Только в диаграммах Эйлера. Все диаграммы Венна являются диаграммами Эйлера, но не все диаграммы Эйлера являются диаграммами Венна. Диаграммы Эйлера имеют только те комбинации пересечений, которые действительно существуют в реальном мире.Диаграммы Венна представляют все гипотетически возможные логические отношения между категориями. Диаграммы Венна по определению должны отображать все возможные комбинации пересечений, что создает некоторые интересные проблемы с компоновкой. Когда есть три круга, отображается каждое пересечение, но как только вы попадаете в четыре категории, круги перестают работать. Эллипсы могут работать до пяти категорий, но помимо пяти необходимо использовать странные формы, чтобы вплетаться и выходить из всех комбинаций пересечений. Пять уже довольно сложно читать, но как только эти странные формы вступают в игру, чтение диаграмм становится почти невозможным, а текстовые описания отношений часто легче понять.Это подводит нас к тому, почему так много забавных диаграмм. Диаграмма Венна позвоночных и беспозвоночных, которую мы показали вам ранее, предполагает, что некоторые животные могут одновременно иметь позвоночник и не иметь позвоночника. Хотя это может быть и не смешно, любой, кто учился в шестом классе по естествознанию, может сказать вам, что это довольно глупо. При более интересном предмете указание на несуществующие отношения может быть довольно забавным. Некоторые диаграммы достигают юмора, помещая что-то в категорию, к которой это не относится, или в категорию, в которой вы обычно не ожидали бы этого.Креативная маркировка (как на диаграмме Эйлера ниже) тоже всегда помогает. Мы знаем, что диаграммы Эйлера могут быть забавными, но могут ли они быть полезными? Некоторые диаграммы включают количественную информацию, используя площадь каждой части диаграммы. Хорошим примером этого является Глобальная карта социальных сетей за 2011 год по данным Global Web Index. Сами по себе диаграммы Эйлера, подобные этой, дают вам довольно хорошее общее представление о задействованных значениях. Считать их точное количество было бы немного сложно, но для получения общего обзора они хороши.На этом конкретном рисунке эти диаграммы Эйлера используются как малые кратные, что позволяет сравнивать регионы. Почему в инфографике нет более серьезных диаграмм Эйлера / Венна? Они хорошо демонстрируют совпадение категорий. Они выглядят «круто» – многие дизайнеры считают это важным. Они разумно демонстрируют количество. Они демонстрируют сложные и интересные отношения в простой для понимания манере. Если вы дизайнер и хотите получить интересные диаграммы, внимательно изучите свои данные. Возможно, вы сможете использовать диаграмму Эйлера или Венна. Дрю Скау – кандидат компьютерных наук, студент-визуализатор в UNCC, со степенью бакалавра архитектуры.

диаграмм Венна с помощью Eulerr

диаграмм Венна с помощью Eulerr
Йохан Ларссон
2021-09-06

Диаграммы Венна – это специализированные диаграммы Эйлера. В отличие от диаграмм Эйлера, они требуют, чтобы были представлены все пересечения. В большинстве реализаций, включая eulerr , они также не пропорциональны площади.

Первое требование часто является запретительным с точки зрения интерпретируемости, поскольку мы часто тратим значительное количество места на холсте диаграммы с пересечениями, которые также могут быть представлены их отсутствием.Диаграмма Эйлера с пропорциональной площадью часто намного более интуитивно понятна и, для относительно редких входных данных, намного легче интерпретируется. Однако свойство пропорциональности площади иногда может быть коварным, по крайней мере, если зритель не осведомлен о недостатках диаграммы.

В таких случаях часто лучше отказаться от пропорциональности площади и использовать диаграмму Венна. Это может быть не так просто для глаз, но, по крайней мере, будет правильно истолковано.

eulerr поддерживает диаграммы только для 5 наборов.Отчасти причина практическая. eulerr построен на основе эллипсов, и эллипсы подходят только для диаграмм Венна, содержащих не более 5 наборов. Другая часть причины связана с удобством использования. Диаграмма из пяти наборов уже расширяет ее с точки зрения того, что, как мы можем разумно ожидать, зритель сможет расшифровать. И, несмотря на веселье в форме банановых диаграмм Венна из шести наборов, таких чудовищ лучше оставить в темноте.

Примеры диаграмм Венна и Эйлера

Теперь мы рассмотрим некоторые диаграммы Венна и соответствующие им диаграммы Эйлера.

  библиотека (eulerr)
set.seed (1)

s2 <- c (А = 1, В = 2)

сюжет (venn (s2))
участок (эйлер (s2), количества = ИСТИНА)  

  сюжет (venn (плод [, 1: 3]))
сюжет (эйлер (фрукты [, 1: 3], shape = "эллипс"), количество = ИСТИНА)  

  s4 <- список (a = c (1, 2, 3),
           б = с (1, 2),
           с = с (1, 4),
           е = с (5))
сюжет (venn (s4))
plot (euler (s4, shape = "ellipse"), sizes = TRUE)  

  участок (венн (организмы))
сюжет (эйлер (организмы, форма = "эллипс"), количества = ИСТИНА)  

Как вы можете видеть на последнем графике, бывают случаи, когда диаграммы Эйлера могут вводить в заблуждение.Несмотря на то, что алгоритм изо всех сил пытается сделать диаграмму пропорциональной по площади, ограничения, накладываемые геометрией эллипсов, препятствуют точному совпадению. Вероятно, это тот случай, когда диаграмма Венна является хорошей альтернативой. По мнению автора, в остальном все наоборот.

Диаграммы и круги Венна

Диаграммы и круги Венна Сатьядев Нандакумар
Вт 17 июл 11:42:14 2018

Диаграммы Венна дают наглядное представление о взаимосвязях между наборы.Мы часто видим диаграммы Венна с кружками, которые изображают отношения, которые могут возникать между 2 наборами или 3 наборами. Например, следующая диаграмма Венна иллюстрирует отношения между 3 наборы.

Можем ли мы нарисовать диаграмму Венна, изображающую все отношения, которые могут существует между 4 наборами с использованием кругов?

Характеристика Эйлера

Эйлерова характеристика геометрической фигуры определяется как V-E + F, где V - количество вершин, E - количество ребер, а F - количество лиц (регионов).

Для любой плоской фигуры характеристика Эйлера равна 2. (Попробуйте квадрат: V = 4, E = 4, F = 2, что дает V-E + F = 2, и треугольник: V = 3, E = 3, F = 2, что дает V-E + F = 2 или следующий рисунок с V = 4, E = 5, F = 3.)

характеристика Эйлера и диаграммы Венна

Любая диаграмма Венна является плоской фигурой, поэтому V-E + F будет 2. Мы можем проверить что на рисунке 1, где V = 6, E = 12, F = 8, что дает V-E + F = 2.

Ключевой момент заключается в следующем: два различных круга могут пересекаться только в ровно 0, 1 или 2 балла.Для диаграмм Венна нам необходимо рассматривать только пары окружностей, которые пересекают друг друга ровно 2 точки.

Чтобы упростить количество случаев, предположим, что каждая отдельная пара кругов соответствует отдельной паре пересечений точки. [сноска]

Попробуем ввести на рисунок четвертый кружок. Четвертый вводимый круг приносит еще 6 вершин. Таким образом V = 12. Каждая вершина имеет степень 4. (рисунок).

Легко установить, что для любого графа сумма степени вершин в два раза больше числа ребер.Следовательно, E = 24. Но поскольку нам нужна плоская фигура, мы должны иметь F = 2-V + E = 2-12 + 24 = 14.

Таким образом, невозможно представить 16 регионов с помощью диаграмм Венна. состоящий из 4-х кружков.

(Эллипсы, овалы, треугольники и т. Д. Могут использоваться для представления отношения между 4 наборами.)

Более 4 комплектов

Количество пар, образованных n кругами, равно n (n-1) / 2. Количество вершин в фигуре пересечения, следовательно, n (n-1), если Venn диаграмма существует как плоская фигура. Количество ребер - 2n (n-1).п = 2.

Поскольку 2 n асимптотически больше n (n-1), единственный два значения n, для которых может выполняться уравнение: n = 2 и п = 3.

Вклад Венна

Однажды в разговоре я и мой друг задавался вопросом заслужил ли Джон Венн бессмертную славу за столь тривиальную концепцию. В в свете приведенного выше обсуждения легче оценить вклад - он дал геометрический схема для изображения отношений между n наборами для произвольного n (!!) Конечно, ему пришлось использовать рекурсивную схему с использованием странных форм.


Сноска: через точку могут проходить 3 круга. Но если отметим, что любая подколлекция из 3 кругов из 4 должна быть действительной Диаграмма Венна для 3-х наборов, можно сделать вывод, что конфигурации, в которых 3 или более круга, проходящие через точку, не имеют значения.

ОБРАТНАЯ СВЯЗЬ: Вас не убедил аргумент? Напишите мне доработки / исправления по адресу [email protected].

Все о диаграмме Венна

Определение диаграммы Венна

Диаграмма Венна (также известная как набор или логическая диаграмма ) может показать различия, сходства и перекрывающиеся отношения между наборами, т.е.е., группы данных. Чаще всего его используют для описания общих черт между противоборствующими фракциями - например, в приведенном ниже примере диаграммы Венна мы можем увидеть общие характеристики клеток растений и животных.

Исходные клетки растений против клеток животных - https://owlcation.com/stem/Plant-Cells-vs-Animal-Cells-With-Diagrams

Набор данных визуализируется как совокупность фиксированных точек внутри замкнутой окружности (обычно кругов, но иногда эллипсов, сфер или даже треугольников): визуально описывает одну категорию, которую имеют все включенные данные.В приведенном выше примере окружность, содержащая характеристики клеток животных, имеет точки данных: неправильная / круглая, центросомы, центриоли, лизосомы, ядро, клеточная мембрана, цитоплазма, митохондрии, аппарат Гольджи и рибосомы. Только , около из этих точек данных / характеристик являются общими для растительных клеток, визуализированных в синем пересечении.

Нравится, как выглядит этот пример диаграммы Венна? Начни и создай свой собственный!

Основная точка интереса диаграмм Венна - это пересечение - данные, которые попадают в более чем одну категорию (в приведенном выше примере это поясняется меткой «обе», хотя следует помнить, что пересечение - это , а не . собственный, отдельный набор данных).Диаграммы Венна могут состоять из нескольких пересечений и наборов кругов, но наиболее часто используется трехкружная или тройная диаграмма Венна.

Одним из факторов популярности тройного Венна является то, что он создает в центре изогнутый треугольник идеальной и постоянной ширины, известный как «треугольник Рело». Эта форма очаровала инженеров, математиков, архитекторов и графических дизайнеров как своими математическими возможностями, так и визуальной красотой, и использовалась эрудитами от немецкого инженера Франца Рёло (в честь которого они теперь названы) для создания кинетических машин и Леонардо Да. Винчи за проектирование карты.

Начните создавать свои собственные диаграммы и бизнес-графику.

Попробуйте Vizzlo бесплатно

Развитие и история диаграммы Венна

В той или иной форме диаграммы в стиле Венна существуют еще со средневековья. Абстрактное визуальное представление соединений и пересечений различных множеств - невероятно старая концепция, поскольку существует глубокое человеческое стремление показать, что принадлежит, а что нет. Как и большинство логических сред, он рос и органически изменялся с годами, уточнялся, перефокусировался и перенаправлялся в круговую диаграмму, с которой мы так хорошо знакомы сегодня.

Первым математиком, напрямую связавшимся с этим стилем диаграмм, был немецкий эрудит и философ Готфрид Вильгельм Лейбниц примерно в конце 1600-х годов. Лейбниц был, по сути, одним из первых приверженцев многих концепций визуализации данных: он верил в символическое представление сложных мыслей, чтобы не только лучше передавать эти идеи, но и оставлять меньше места для человеческих ошибок, неверной интерпретации и возникающих споров. Для Лейбница этого удалось добиться путем вычисления и визуализации разногласий в формализованных логических диаграммах.

Диаграмма Эйлера

Примерно сто лет спустя знаменитый швейцарский математик Леонард Эйлер стал широко известен тем, что использовал перекрывающиеся круги для представления логических выводов (или силлогистических рассуждений). Эти круги используются для изображения четырех категоричных положений силлогизма:

  1. Универсальное подтверждение «Все графики Vizzlo выглядят красиво»
  2. Универсальный отрицательный «Ни один из графиков Vizzlo не является трудным для создания»
  3. Особо утвердительное «Некоторые из Vizzard Vizzlo включают диаграммы Венна»
  4. Особо отрицательный «Некоторые из Vizzard Vizzlo не являются диаграммами Венна»

Заметка на полях: Знаете ли вы, что в соответствии с логической семантикой в ​​английском языке до конца XVI века было четыре слова «да» и «нет» ?! Универсальное Утверждение было бы сказано как «Да», Универсальное Отрицательное как «Нет», Частное Утверждение как «Да» и Частное Отрицательное как «Нет»."

Чтобы было ясно, стало обычной практикой данных, а именно: группировать диаграммы Эйлера и Венна под более известным названием «диаграмма Венна», чтобы избежать путаницы - только в очень технических математических ситуациях может потребоваться разграничение между ними.

Современная диаграмма Венна

Диаграмма Венна, известная нам сегодня, была разработана английским математиком Джоном Венном.В 1880 году он опубликовал статью под названием «О схематическом и механическом представлении суждений и рассуждений». В нем Венн предложил изменить диаграмму Эйлера, чтобы она стала более всеобъемлющей визуализацией вероятности, потребовав отображения всех возможностей. Однако название «диаграмма Венна» было придумано американским философом Кларенсом Ирвингом лишь несколько лет спустя, в 1918 году.

С тех пор диаграмма Венна стала центром обширных исследований многих других известных ученых, в том числе А.W.F. Эдвардс, Бранко Грунбаум и Генри Джон Стивен Смит. Они стали важной визуализацией для математических исследований, будь то традиционная трехкружная диаграмма или более сложные версии, включая сферы и эллипсы, для таких областей, как теория множеств, теория логики, вращательная симметрия и изучение сложных форм и чисел, таких как тессеракт.

Что НЕ является диаграммой Венна?

Для такой простой диаграммы Венна с годами неправильно обозначали. Фактически, сейчас мы чаще всего обращаемся почти исключительно к диаграммам Эйлера, когда обсуждаем «Венн.”

Диаграммы Венна по сути являются более специфическим типом диаграмм Эйлера. Вот удобная диаграмма Эйлера, наглядно описывающая, что все диаграммы Венна являются диаграммами Эйлера, но не все диаграммы Эйлера являются диаграммами Венна:

Золотое правило Венна состоит в том, что он всегда визуализирует всех возможностей, даже если ни один из данных не укладывается в эти пересечения, тогда как диаграмма Эйлера визуализирует только категории, для которых есть данные. Если бы мы заставили приведенную выше диаграмму соответствовать параметрам диаграммы Венна, мы должны были бы представить себе, что существуют некоторые диаграммы Венна, которые не являются диаграммами Эйлера.Чтобы поместить это в контекст, это похоже на утверждение, что некоторые животные не пингвины, и поэтому некоторые пингвины не являются животными.

Чтобы продемонстрировать это различие на практике, представим, что трое друзей сравнивают свои домашние задания. Люси подготовила два домашних задания: одно по теме «Прокариоты и эукариоты» по естествознанию и одно по теме «Вероятность и статистика» по математике. Анвар также подготовил домашнее задание по математике на тему «Вероятность и статистика», но выбрал для науки тему «Клетки животных и клетки растений».Янс написал отличное эссе на тему «Митоз и мейоз» для Science, но совершенно забыл, что его домашняя работа по математике вообще должна была быть.

Если оранжевый набор данных представляет Янса, розовый набор данных представляет Анвара, а синий набор данных представляет Люси, мы можем организовать эту информацию двумя разными способами.

Диаграмма Эйлера показывает всего возможностей, по которым у нас уже есть фактические данные. Мы знаем, что Анвар и Люси выполнили домашнее задание по математике на тему «Вероятность и статистика», и пересечение показывает это.Мы знаем, что Янс не создавал никаких домашних заданий по математике, как Анвар и Люси, и что они не выбрали ту же тему, что и он, для науки, поэтому набор данных Янса не пересекается с другими.

Диаграмма Венна показывает ту же информацию - темы, которые трое друзей изучали индивидуально, и математическую тему, которую выбрали Анвар и Люси. Однако у нас также есть пересечения, которые не содержат данных, но представляют всех других возможностей.

Оранжевое пересечение слева представляет собой случай, когда Янс вместе с Анваром помнит свою домашнюю работу по математике, а также науку, но в этом гипотетическом результате Люси забывает завершить работу по математике.Зеленый перекресток справа олицетворяет ту же судьбу Анвара. Центральное пересечение показывает вероятность того, что все трое друзей не забудут работать над «Вероятностью и статистикой» для математики, а также над различными темами для естествознания, и никто не получит наказание…

Готовы создать свою собственную диаграмму Венна? Вот небольшой ярлык для страницы Vizzard специально для вас.

Для построения диаграммы Венна между двумя наборами данных должны быть как общие черты, так и различия.Если вся информация по категориям идентична, ваша диаграмма будет выглядеть так:

Даже с двумя наборами данных, если содержащаяся информация не имеет различий, две кривые будут полностью перекрываться, создавая только один круг.

Противоположная проблема, два набора данных без общих черт, также не будут создавать диаграмму Венна (но вполне приемлемую диаграмму Эйлера, показывающую непересекающихся множеств ).

Удивительно, но то, что выглядит столь же знакомым, как приведенный ниже пример, также математически не считается диаграммой Венна.

Как объяснялось ранее, одним из ключевых требований диаграммы Венна является то, что она должна отображать все возможные логические результаты. Поскольку в приведенном выше примере нет пересечения, в котором встречаются только желтый и красный или только синий и оранжевый, это не может считаться полным отображением универсальной логики.

Сводка

Подводя итог, диаграмма Венна:

  • Тип диаграммы Эйлера
  • Диаграмма с двумя или более наборами данных, изображенными в пределах окружности
  • Используется для визуализации различий и общих черт между этими наборами данных
  • Используется для выработки всех логических выводов
  • Невероятно быстрое изготовление с использованием диаграммы Венна Vizzlo Vizzard

Vizzlo очень проста, и вы можете попробовать ее бесплатно.

Начните создавать свои собственные диаграммы и бизнес-графику.

Попробуйте Vizzlo бесплатно

Что такое диаграммы Венна в математике? - История, типы и примеры - Видео и стенограмма урока

Типы диаграмм Венна

Диаграммы Венна полезны для иллюстрации нескольких типов отношений.

Непересекающиеся множества

На примере урока естествознания и математики из предыдущего, начальная диаграмма представляет непересекающиеся множества, потому что два множества (урок естествознания и музыкальный класс) не имеют ничего общего.Учащиеся музыкального класса находятся только в вашем музыкальном классе, а ученики в классе естественных наук - только в вашем классе. Между ними нет никакой связи. Непересекающиеся множества всегда изображаются отдельными кружками.

Пересечения

Если взять предыдущий пример диаграммы Венна с непересекающимся множеством, то если вы перекрываете круги, это указывает на пересечение. Пересечение представляет элементы, которые есть в обоих наборах.Он представляет собой общность между наборами. Итак, в примере с классом музыки и естествознания пересечение обозначено серым цветом и означает, что Тайлер и Лео посещают и ваши музыкальные, и естественные уроки.

Пересечение обозначено серым цветом.

Перекресток образует еще один набор. Он может быть обозначен письменно перевернутым U и будет выглядеть так:

Заштрихованную серую область, включая Тайлера и Лео, можно записать так, перевернутую букву U, указывающую на пересечение обоих наборов.

Если мы вернемся к примеру двух непересекающихся множеств с двумя отдельными кругами, они не будут образовывать пересечения. Это называется пустым набором и может быть записано так:

Окружность, через которую проходит наклонная линия, означает, что существует пустое множество из-за непересекающихся множеств.

Объединения

Объединение - это набор элементов, содержащихся в обоих наборах, включенных только один раз.Давайте посмотрим на диаграмму Венна с перекрывающимися наборами A , B и C .

Диаграмма Венна с перекрывающимися множествами A, B и C.

Мы бы записали это объединение, включив каждый элемент из A , B и C только один раз, даже если в наборах есть повторяющиеся элементы. Это будет выглядеть так:

Каждый элемент включается только один раз при написании союза.
Дополнения

Дополнения - это способ сказать «все, чего нет». Обозначается маленькой цифрой c . Используя диаграмму Венна из раздела объединений, если мы хотим показать «все, чего нет в A », мы могли бы заштриховать все остальные области, кроме A .

Эта диаграмма Венна представляет все, что не находится в A, заштрихованным серым цветом.

Эта диаграмма Венна, представляющая Дополнение до , может быть записана следующим образом:

Подмножества

Диаграммы Венна могут использоваться для обозначения подмножеств.Если маленький круг находится внутри большего круга, можно сказать, что меньший круг включает в себя свойства большего круга и, таким образом, является подмножеством большего круга. На этой иллюстрации большой круг охватывает всех животных с крыльями. Меньший круг - это динозавры с крыльями, которые представляют собой подмножество животных с крыльями. Большой круг на самом деле является подмножеством универсального множества, которым являются «животные».

Диаграммы Венна и задачи со словами

Мэри нужно сделать снежные шишки для большинства учеников ее класса из 50 (включая ее) во время полевого дня в средней школе.У нее всего два вкуса: вишня и голубая малина. 30 студентов хотят снежный конус с вишневым вкусом, а 20 хотят снежный конус с ароматом голубой малины, но 15 из этих студентов хотят снежный конус с вишневым и синим малиновым вкусом. Сколько всего учеников хотят снежный конус?

В этом примере мы нарисовали бы диаграмму Венна с двумя кружками внутри прямоугольника. Прямоугольник означает, что в ее классе 50 учеников. Первый кружок указывает на вишневый вкус.Второй кружок указывает на вкус голубой малины. Общая область кружков будет указывать на студентов, которые хотят ароматизировать как вишню, так и голубую малину. Поскольку 15 из 30 учеников, которые хотят снежный рожок с вишневым вкусом, хотят и вишневого, и синего малинового, вы должны вычесть 15 из 30, получая 15 детей, которым просто нужен снежный рожок со вкусом вишни. Поскольку 15 из 20 учеников, которые хотят снежный конус со вкусом голубой малины, хотят и вишню, и голубую малину, вы должны вычесть 15 из 20, получив 5 детей, которым просто нужен голубой снежный конус со вкусом малины.В конце концов, вы добавляете единственных учеников-вишенок к единственной голубой малине и добавляете их к ученикам как вишневых, так и голубых малиновых, чтобы увидеть, сколько всего учеников хотят снежный конус, а это будет 35 из 50 детей.

Резюме урока

Диаграммы Венна были созданы Джоном Венном в 1880 году, хотя они не назывались диаграммами Венна, пока Кларенс Льюис не назвал их так в своей книге, опубликованной в 1918 году.Это круги, которые показывают общие черты и различия между двумя или более наборами. Набор - это группа элементов, обозначенных {скобками}. Существует пять типов диаграмм Венна. Диаграмма называется непересекающейся , если нет общих черт, то есть есть два отдельных круга. Перекрестки - это места, где круги перекрываются, чтобы показать общность. Объединения - это наборы элементов, содержащиеся в обоих наборах, но включаемые только один раз, то есть вы записываете их только один раз, даже если они появляются несколько раз. Дополнения показывают все, что не является определенным набором. Наконец, подмножеств показывают набор, который является частью большего набора.

Что такое диаграмма Венна? (С примерами)

Ключевые выводы:

  • Диаграммы Венна используются математиками, учителями и другими людьми, которым необходимо легко представить сложные данные. Обычно они состоят из двух-трех кругов, которые перекрываются для сравнения.
  • Диаграммы Венна бывают трех типов.Чтобы создать его, установите параметры вашего анализа, создайте свою вселенную, пометьте свои наборы, разместите свои данные и затем проанализируйте их.
  • Диаграммы Венна имеют несколько преимуществ, в том числе позволяют людям визуализировать информацию, а также понимать и запоминать сложные темы.

Диаграмма Венна - полезный инструмент для математиков, учителей, статистиков и всех, кому необходимо представить визуальные эффекты или упростить понимание сложной информации. В этой статье мы объясним, что такое диаграмма Венна, ее приложения и преимущества, а также как создать свою собственную диаграмму Венна.

Что такое диаграмма Венна и для чего она используется?

Диаграмма Венна - это представление того, как группы связаны друг с другом, с группами, которые называются «множествами». Диаграммы Венна обычно состоят из двух-трех кругов или других фигур, которые перекрываются, но в зависимости от количества наборов на диаграмме может быть больше фигур.

Каждая фигура представляет собой набор чисел, предметов или понятий. Когда наборы имеют похожие значения, эти значения появляются в перекрывающихся областях, называемых «пересечениями»."

Диаграммы Венна названы в честь английского логика Джона Венна. Эти формы в значительной степени основаны на диаграммах Эйлера, но, в отличие от диаграмм Эйлера, диаграммы Венна предназначены для отображения всех возможных отношений между двумя или более анализируемыми группами, даже если одна или несколько наборов не имеют значений.

Связанное: Что такое визуализация данных: определение, типы и инструменты

Как создать диаграмму Венна

Вы можете создать диаграмму Венна, выполнив следующие действия:

1 .Установите параметры вашего анализа

Сначала определите, какова ваша цель, например, принятие важного решения или понимание сложных тем. Затем определите, сколько понятий или наборов идей, чисел или объектов будет задействовано. Затем определите тип диаграммы Венна, которую нужно создать, установив, как вы хотите показать взаимосвязь каждого набора друг с другом.

2. Создайте свою вселенную

Когда вы узнаете, что вы хотите делать со своей диаграммой Венна, создайте область или «вселенную», которая обычно представляет собой прямоугольник, в котором будут располагаться все остальные формы.Убедитесь, что другие формы перекрываются, чтобы вы могли найти возможные отношения между наборами.

3. Обозначьте все свои наборы

Выбранные имена наборов должны отражать простые или упрощенные идеи, которые относятся ко всем данным в каждом наборе. Например, если вы сравниваете двух персонажей из романа, вы должны использовать имена персонажей для обозначения наборов. В математике наборы обычно имеют отдельные буквы (например, A, B, C и т. Д.), Которые служат метками для наборов.

4.Поместите данные на диаграмму Венна

Если вы хотите проанализировать все данные, поместите их на диаграмму на основе соответствующего набора. Если данные из одного набора встречаются на пересечении с другим, запишите эту информацию только один раз, на пересечении.

5. Проанализируйте данные

Найдите аналогичную информацию в каждом наборе, взвесьте различия и сделайте выводы. Если вы пытаетесь принять важное решение, различия, которые вы обнаружите между группами, помогут вам прийти к такому выводу.

Связанные: 7 эффективных методов анализа данных

Какие три типа диаграмм Венна?

Диаграммы Венна помогают выражать идеи или анализировать данные. Вот три основных типа:

Диаграммы Венна

Когда Венн задумал свою диаграмму, он начал с двух симметричных замкнутых кривых (кругов), но допустил создание диаграмм с тремя или более формами:

  • Два круга диаграмма. Двухкружная диаграмма Венна показывает взаимосвязь между двумя наборами информации.Два круга обычно пересекаются по вертикали, поэтому одна фигура находится слева, а другая - справа.
  • Трехкружная диаграмма. Его трехкружная диаграмма показывает взаимосвязь между тремя группами информации. Обычно два набора ориентированы так, как если бы были доступны только два набора информации, а третий набор пересекает два других по горизонтали (вверх или вниз).
  • Четырехкружная диаграмма. Его четырехугольная диаграмма потеряет симметрию, и пересечение всех форм станет возможным.Венн задумал схему из четырех фигур, которая включает три круга и изогнутую форму. В качестве альтернативы он построил диаграмму с четырьмя эллипсами.

Помимо четырех наборов, Венн построил наборы, в которых каждая последующая кривая будет переплетаться с предыдущими кривыми, пересекаясь с исходными тремя наборами.

Диаграмма Бранко Грюнбаума

Математик Бранко Грюнбаум создал диаграмму Венна из пяти наборов:

  • Диаграмма содержит конгруэнтные, осесимметричные эллипсы
  • Все формы пересекаются на большом пересечении
    1. 25 Всего на диаграмме Бранко Грюнбаума Венна 31 регион.Некоторые математики экспериментируют с ним и помещают числа во все области так, чтобы сумма во всех пяти наборах (A, B, C, D и E) была одинаковой.

      Диаграммы Эдвардса Венна

      Британский статистик Энтони Уильям Фэйрбэнк Эдвардс создал серию диаграмм, которые составляли три или более наборов:

      • Диаграмма с тремя наборами. Его трехкомпонентная диаграмма состоит из двух прямоугольников, пересекающих круг под прямым углом (один прямоугольник ориентирован горизонтально, а другой - вертикально).
      • Четырехнаборная диаграмма. Его диаграмма из четырех наборов включает форму, напоминающую шов теннисного мяча, и расположенную вокруг центра круга. По длине форма шва тянется вертикально.
      • Диаграмма из четырех комплектов плюс. Помимо четырех наборов, Эдвардс допустил последовательные формы зубчатых колес, каждая из которых удваивает количество зубцов по сравнению с предыдущей формой. Каждая из этих форм покоится в центре вселенной.

      Связанные: 10 логических вопросов на собеседовании, которые могут возникнуть в ходе собеседования (с ответами)

      Когда использовать диаграмму Венна

      Диаграммы Венна часто встречаются в математическом контексте, но предприятия и профессионалы также их используют .В каждом случае человек, создающий иллюстрацию, хочет решить проблему, принять важное решение, предсказать вероятности или понять или визуализировать, как несколько наборов, концепций или объектов связаны друг с другом. Примеры, когда диаграмма Венна может быть полезной, включают:

      Математика

      Математические диаграммы Венна позволяют ученым решать сложные задачи, такие как:

      • Задача объединения: Задача объединения требует, чтобы учащиеся поместили все числа вместе наборы на схеме.
      • Задача пересечения: Задача пересечения требует, чтобы учащиеся размещали только те числа, которые пересекаются на диаграмме. Например, если задача дает набор из трех чисел (1, 15 и 27) и набор из семи чисел (1, 3, 14, 19, 21, 25 и 27), на диаграмме появятся только 1 и 27. , на пересечении кругов.
      • Симметричная проблема: Симметричная разность двух наборов требует, чтобы на иллюстрации были представлены только числа, которые не перекрываются.
      • Абсолютная проблема: Абсолютное дополнение одного набора требует, чтобы все числа, не входящие в этот набор, были представлены на диаграмме.

      Связанные: дедуктивное рассуждение: определение и примеры

      Business

      Диаграммы Венна используются в нескольких бизнес-примерах, особенно как часть слайд-шоу. Вот и другие способы:

      • Анализ рынка: При использовании двух или более наборов информации перекрывающиеся области содержат целевой рынок бизнеса.
      • Анализ конкурентов: Сравнивая себя с конкурентами, компания может использовать только два набора информации, чтобы увидеть, чем они отличаются или похожи. Это помогает выявить преимущества и сосредоточиться на улучшениях.
      • Сравнение продуктов: Иллюстрация с перекрывающимися формами может помочь взвесить преимущества двух или более идей продукта, чтобы определить, какие особенности продукта являются наиболее желательными, как показано в перекрывающихся областях.
      • Принятие решений: Те же принципы анализа двух или более продуктовых идей применимы к общему процессу принятия решений в бизнесе.

      Связано: Как анализ данных может улучшить процесс принятия решений

      Другое применение

      Диаграммы Венна также используются в информатике, лингвистике, логике, статистике и обучении, в том числе:

      • Визуализация компьютерных языков и их иерархии
      • Понимание сходства разных языков
      • Работа с логическими операторами, которые содержат союзы «или» и «и», для проверки достоверности этих аргументов
      • Сравнение возможных событий и прогнозирование вероятности их появления
      • Улучшение понимание прочитанного, например, при анализе романов, чтобы понять различия и сходства между персонажами, идеями, темами и настройками.

Добавить комментарий

Ваш адрес email не будет опубликован. Обязательные поля помечены *